Välkommen till Resurscentrums frågelåda!

 

Vill du ha ett snabbt svar - sök i databasen: Anpassad Google-sökning
(tips för sökningen).
Använd diskussionsforum om du vill diskutera något.
Senaste frågorna. Veckans fråga.

63 frågor/svar hittade

Ljud-Ljus-Vågor [19419]

Fråga:
Hur får föremål färg?
/Veckans fråga

Ursprunglig fråga:
Hej! Vi läser just nu optik i skolan o lär oss om ljuset. Vi har pratat mycket om ljusets frekvens och att de olika frekvenserna ger olika färger. Men när man upplever att ett föremål har en viss färg så reflekterar föremålet det vita ljuset och absorberar komplementfärgen. Vad innebär det? Absorberas alla färger utom just den vi upplever eller bara komplementfärgen? Skulle vara kul att få veta! Mvh Mirjam
/Mirjam O, Vallhallaskolan, Oskarshamn

Svar:
Hej Mirjam! Bra fråga!

Komplementfärg är ett besvärligt och dåligt definierat begrepp, se fråga 17926 . Det är dessutom kopplat till hur människan uppfattar färgblandningar. Därför är det lättare att förstå vad som händer i termer av våglängder. Våglängderna är sedan kopplade till känsligheten hos röd-, grön- och blåkänsliga tappar i ögat, se fråga 13824 .

Om ett föremål som belyses av vitt ljus (alla synliga våglängder) absorberar alla våglängder utom de röda som reflekteras, så uppfattas föremålet som rött.

Klorofyll t.ex. absorberar blått och rött (se fråga 10888 ), så det reflekterade gröna ljuset ger växtbladen sin gröna färg.

Vad händer om grönt ljus absorberas och rött och blått reflekteras? Det beror alltså på tapparnas känslighet för olika våglängder. Man kan blanda färger med appleten Color Addition Simulator . Där kan man se att blandningen rött/blått uppfattas som en färg vi kallar magenta. Observera att denna färg inte finns i spektrum utan är en färg som uppstår vid blandning av flera färger i kombination med ögats färgkänslighet.
/Peter E

Nyckelord: färg/färgseende [39]; #ljus [63];

*

Ljud-Ljus-Vågor [19387]

Fråga:
Varför är klorofyllet inte svart?
/Veckans fråga

Ursprunglig fråga:
Hej, jag undrar varför klorofyllet inte är svart? Jag menar, växterna vill ju ta upp så mycket solljus som möjligt, så varför är då inte växter svarta? Att växterna vid ekvatorn inte är svarta är förståeligt. De lär ju brinna upp, men här in norden är det ju brist på solljus, så varför har då inte växterna här svart klorofyll? Har det något med evolutionen att göra? Och om man då på konstgjort sätt skapar en växt med svart klorofyll, skulle denna växt då kunna konkurrera ut andra växter? / Johanna
/Johanna L, Bjärehovskolan

Svar:
Man kan tycka att det skulle vara mest effektivt om klorofyllet absorberade alla våglängder och inte bara blått och rött, se fråga 10888 . Nu fungerar emellertid inte evolutionen så att den effektivaste lösningen alltid utvecklas. För detta krävs dels en uppenbar fördel som driver evolutionen och dels att lösningen är fysikaliskt möjlig.

I fallet fotosyntes hos växter är det en mycket komplex process som antagligan bara utvecklats en enda gång med grönalger (Grönalger ) som gemensam förfader. Sedan måste det även finnas en molekyl som till skillnad från klorofyll absorberar även grönt ljus. På frågan "varför har då inte växterna här svart klorofyll?" är svaret att svart klorofyll finns inte. Jag kan tänka mig att man skulle kunna hitta något ämne som absorberar grönt ljus och som kunde komplettera klorofyll (se länk 2). Med hjälp av genteknik skulle man kanske kunna få växter att producera den grönabsorberande molekylen. Det finns emellertid andra metoder att öka effektiviteten, se nedan.

Se även diskussionen i länk 1, länk 2 och
http://simple.wikipedia.org/wiki/Chlorophyll#Why_green_and_not_black.3F

I fråga 10888 finns även en video om hur man kan optimera belysningen i växthus genom att med LED generera endast de våglängder klorofyllet absorberar.
/Peter E

Nyckelord: *biologi [20]; Darwins evolutionsteori [8]; genteknik [2]; #ljus [63];

1 http://www.askabiologist.org.uk/answers/viewtopic.php?id=3837
2 http://hyperphysics.phy-astr.gsu.edu/hbase/biology/pigpho.html

*

Ljud-Ljus-Vågor [19128]

Fråga:
Hur fungerar en självfokuserande kikare?
/Veckans fråga

Ursprunglig fråga:
Hur fungerar en självfokuserande kikare? Hur kan kikaren ställa in sig själv efter personer med olika syn (synfel)?
/Anna G, Stenungsund

Svar:
Intressant fråga - det går naturligtvis inte! Autofokus för kikare är bara en marknadsföringsploj, se länk 1. I själva verket är kikarna med autofokus konstruerade så att de fokuserar på oändligt avstånd och har stort skärpedjup. Till en del kan även ögats egna autofokusering korrigera för ändligt avstånd genom att ögonlinsen ändrar form.

För att åstadkomma riktigt autofokus behöver systemet ha en inbyggd detektor. I elektroniska kameror finns ju detta. Det finns flera sätt att åstadkomma autofokus. Det enklaste är att maximera kontrasten - gränslinjen mellan ljust och mörkt är skarpast om man har korrekt fokusering. En annan metod är att använda ultraljud eller infrarött ljus. Se vidare Autofocus och länk 2.
/Peter E

Nyckelord: teleskop [10]; ögat [18]; #ljus [63];

1 http://www.bestbinocularsreviews.com/self_focusing_binoculars.php
2 http://electronics.howstuffworks.com/autofocus.htm

*

Ljud-Ljus-Vågor [18817]

Fråga:
Frågan gäller diffus reflexion: Gäller reflexionslagen vid diffus reflexion? Om lagen inte gäller undrar jag varför.
/Per W, Fäladsgården, Lund

Svar:
Hej Per!

Nej, reflektionslagen gäller inte för hela ytan som en spegelyta där reflektionsvinkeln r är lika med infallsvinkeln i.

Reflektionslagen kan härledas från Fermats princip: ljuset tar den snabbaste vägen (se fråga 11135 och länk 1). Ett annat sätt at se det är att den infallande ljuset genererar svängningar hos laddningar. Dessa svängningar ger upphov till elektromagnetisk strålning för vilken alla riktningar utom r=i interfereras bort - Huygens princip, se fråga 2417 . Se vidare Reflection_(physics) .

Diffus reflektion: Ljuset reflekteras i alla tänkbara vinklar på grund av små ojämnheter i ytan. Detta gäller alla matta ytor som inte är svarta.

För diffus reflektion har du alltså inte en plan yta utan en massa ytor som är riktade åt olika håll. Reflektionsvilken blir då slumpmässig, se nedanstående figur från Wikimedia Commons. Fördelningen av det reflekterade ljuset beror på det reflekterande materialets struktur, Se Diffuse_reflection .



/Peter E

Nyckelord: ljusreflektion [18]; #ljus [63];

1 http://hyperphysics.phy-astr.gsu.edu/hbase/phyopt/fermat.html#c2

*

Ljud-Ljus-Vågor [17944]

Fråga:
Varför är fjärrkontrollens IR-ljus lila i en mobiltelefonkamera?
/Veckans fråga

Ursprunglig fråga:
Om man tar ett kort av en fjärrkontroll med en digitalkamera samtidigt som man trycker på någon av knapparna på fjärrkontrollen så kan man se det infraröda ljuset som den använder. Detta ljus ser då lila ut på kamerans skärm.

Varför tolkar kameran ljuset som lila? Lila ljus har ju en väldigt kort våglängd och ligger långt bort från det infraröda ljuset. Borde inte kameran tolka ljuset som rött, det ligger ju närmare det infraröda ljuset i våglängd?
/Jenny O, Carl Swartzgymnasiet, Norrköping

Svar:
Bra fråga Jenny! Det kan tyckas konstigt att infrarött kommer ut som lila - det hade varit mer naturligt med rött som är grannfärgen. Det beror helt enkel på hur kameran är konstruerad. För att detektera färger har man tre CCDer i varje bildpunkt med ett rött, grönt och ett blått filter. Dessa filter fungerar bra i synligt ljus, så att färgåtergivningen blir korrekt. Det är uppenbarligen så att det är det röda och det blå filtren som släpper in mest infrarött, så att resultatet blir lila, se nedanstående bild.

Se även CCD_camera#Architecture och länk 1 sidan 29.



/Peter E

Nyckelord: färg/färgseende [39]; #ljus [63];

1 http://www.fysikersamfundet.se/Fysikaktuellt/2015_1.pdf

*

Ljud-Ljus-Vågor [17926]

Fråga:
När det gäller färgerna har jag förstått att vi uppfattar de färger som inte absorberas av det ämnet ljuset reflekteras på tex en röd tröja. Alla våglängder utom röd absorberas. Men nu har jag ett nyskrivet läromedel som skriver att det bara är komplementfärgen grön som absorberas och att alla andra våglängder studsar tillbaka och tex till ögat. Är det då mängden av andra våglängder som avgör tonen på den röda tröjan ?
/Patrik H, Vasaskolan

Svar:
Den färg vi uppfattar att en yta har beror dels på vilka våglängder som reflekteras och dels på hur det mänskliga ögat uppfattar olika våglängder.

Komplementfärg kan man säga är en färgs motsatsfärg. I vissa färgcirklar ligger dessa färger mitt emot varandra, se nedanstående figur. När man blandar dessa två motsatsfärger uppstår en neutralisead ton (grå) (se Komplementfärg och Complementary_colors ).

Komplementfärg och hur en blandning av färger uppfattas är beroende av att ögan har stavar för 3 olika färger (rött, grönt och blått), se fråga 13824 . Detta svar innehåller länkar till information om färgblandning. Se vidare fråga 16135 och 10888 .

På grund av sättet som vi uppfattar färger kan man åstadomma samma färg på olika sätt. Låt oss ta gult som ett exempel. Natrium har en dubbel emissionslinje i det gula området av spektrum (fråga 19445 och Visible_spectrum ). Men man kan även åstadkomma gult genom att blanda rött och grönt, se appen Color Addition Simulator .



/Peter E

Nyckelord: färg/färgseende [39]; #ljus [63];

*

Ljud-Ljus-Vågor [17691]

Fråga:
Har ljusets olika färger olika hastighet?
/Veckans fråga

Ursprunglig fråga:
Har ljusets olika färger olika hastighet?

Enligt fråga [17367] så får jag en liten förklaring till varför ljusets bryts när det träffar ett tätare medium.

Det som jag inte blir klok på är att det i fysikböckerna (delvis förenklat) står att detta beror på att ljusets hastighet ändras, att det sänks när det träffar det tätare mediet. Ju långsammare ljuset går desto större blir brytningen.

Eftersom ljuset färger bryts olika så säger den principen då att de olika färgerna har olika hastigheter genom ett tätare medium. Bör detta då inte betyda att det gäller för alla olika medium? Tar vi det ett steg längre borde det violetta ljuset, som bryts mest, ha den lägsta hastigheten och därför nå jorden allra sist, om vi tittar på en specifik partikel eller vågrörelse som lämnar solen.

Hur ska vi förhålla oss till detta med våra elever som gärna ställer invecklade frågor men som har svårt att förstå en del fakta.
/Petri M, Mariefreds skola, Mariefred

Svar:
Brytningsindex (Refractive_index , brytningsindex ) n eller optisk täthet hos ett medium definieras som ljushastigheten i vakuum dividerat med ljusets hastighet i mediet v: n = c/v.

I ett medium med brytningsindex n>1 har olika våglängder olika hastighet. Hastigheten är c/n där c är ljushastigheten i vakuum. Observera emellertid att hastigheten i vakuum är c för alla våglängder eftersom n=1.

Ja det är korrekt att ju långsammare ljuset går (ju större n) desto mer brytning får man, se fråga 17367 och 3302 .

Eftersom ljushastigheten i vakuum alltid är densamma så uppkommer inte problemet att vi ser olika tider i olika våglängder. Om rymden mellan jorden och solen varit av glas, så hade vi sett rött ljus snabbare är blått.

Den klassiska förklaringen för ljusbrytning (refraktion, Refraktion ) är en rad soldater som marscherar snett in mot en rak gränslinje till en leråker. Soldaternas marschhastighet minskar när de kommer till leråkern. För att bevara den snygga räta linjen med soldater bredvid varandra, så måste de ändra riktning lite mot normalens riktning. Om vi sedan har en rad med kortbenta soldater som har ännu mindre hastighet i leråkern, så behöver dessa avvika ännu mer från utgångsvinkeln. (Vi vill bara inte tänka på vad som skulle hända om vi blandade långbenta och kortbenta soldater i en rad .)

OK med soldater som kan bestämma att de behöver avvika från rakt fram, men hur vet ljuset hur det skall avvika? Helt enkelt genom att vågor i oordning tenderar att släcka ut varandra (interferens). I alla riktningar utom den som specificeras av brytningslagen släcks alltså ljuset ut. Se bilden nedan.

Din sista fråga finns behandlad i fysik, förståelse av .

Huvudpunkterna är:

  • Utgå från observationer - fysik är inte matematik utan en empirisk (erfarenhetsbaserad, Empiri ) vetenskap
  • "Varför?" kan vi aldrig besvara eftersom det implicerar avsikt (och i så fall, vems avsikt). Man kan emellertid besvara "hur?" och ge samband mellan olika fysikaliska fenomen
  • En del av den teoretiska fysiken är helt enkelt svår att förstå, så vi får lita på vad våra vänner teoretikerna säger åtminstone vad gäller väl etablerade fenomen

Man kan heller inte begära att en lärare skall kunna besvara allt. Även jag, med 40 års erfarenhet av fysik, går bet på en del frågor (ofta de som verkar triviala till att börja med). Jag kan emellertid oftast förstå när jag slår upp ett fenomen, men att göra det lättbegripligt för var och en kan vara svårt eller omöjligt. Det är en del i läroprocessen att man med tiden och erfarenheten förhoppningsvis får en allt djupare förståelse för fysik - allt kan inte komma på en gång! Det är lätt att lära sig att Gustav II Adolf dog en novemberdag 1632 på ett fält nära Lützen, men kanske lite längre att förstå vad han hade där att göra!

Se en avancerad framställning av problemet i länk 1.

En kommentar om förståelse av fysik (Jocelyn_Bell ):
(Jocelyn Bell är en av upptäckarna av pulsarer.)

Jocelyn Bell was born in Belfast, Northern Ireland, where her father was an architect who helped design the Armagh Planetarium. She was encouraged to read and drawn to books on astronomy. She lived in Lurgan as a child and attended Lurgan College where she was one of the first girls there who was permitted to study science. Previously, the girls' curriculum had included such subjects as cross-stitching and cooking.

At age eleven, she failed the 11+ exam and her parents sent her to the Mount School, York, a Quaker girls' boarding school. There she was impressed by a physics teacher, Mr. Tillott, who taught her:

You don't have to learn lots and lots ... of facts; you just learn a few key things, and ... then you can apply and build and develop from those ... He was a really good teacher and showed me, actually, how easy physics was.



/Peter E

Nyckelord: ljusbrytning [26]; fysik, förståelse av [17]; ljushastigheten [24]; #ljus [63];

1 https://thesis.library.caltech.edu/6594/3/Yura_ht_1962.pdf

*

Ljud-Ljus-Vågor [17478]

Fråga:
Varför är vanligt fönsterglas transparent för UV-A, men släpper inte igenom kortare våglängder som UV-B och UV-C?
/Veckans fråga

Ursprunglig fråga:
Varför är vanligt fönsterglas transparent för UV-A, men släpper inte igenom kortare våglängder såsom UV-B och UV-C?
/ulrika e, risbergskaskolan, örebro

Svar:
Ulrika! Min favorifråga, varför? Svaret är som alltid: så är naturlagarna! Det är helt enkelt en egenskap hos glas att det är transparent för synligt ljus och för närliggande UV-A medan det absorberar de mer kortvågiga UV-B och UV-C. Genom att titta på problemet i ett mikroskopiskt perspektiv kan man emellertid säga lite om varför.

Vad är det som gör att ett material absorberar elektromagnetisk strålning av en viss energi? Fria atomer absorberar bara vissa våglängder (spektrallinjer), se fråga 176 . Vilka våglängder som absorberas beror på energinivåerna i atomerna och dessa beror på atomstrukturen. Atomstrukturen i sin tur beror på hur atomen är sammansatt, dvs kärnans laddning (som bestämmer vilket grundämne det är frågan om) och kvantmekanikens lagar.

Om det absorberande materialet är molekyler får man absorptionsband i stället för linjer. Orsaken är att man för molekyler även kan ha vibrations- och rotationstillstånd.

I vätskor och fasta ämnen är atomerna mycket nära varandra vilket medför att energinivåerna blandas ihop så man får kontinuerliga band av absorption. Det är detta som orsakar att olika material har olika färger (se t.ex. fråga 10888 ).

Det finns ju många olika sortes glas, en del som är färgade och alltså innehåller atomer som absorberar i synligt ljus. Fönsterglas är emellertid transparent för våglängder ned till och med UV-A, se nedanstående figur. Orsaken är helt enkelt att de tillgängliga energinivåerna ligger så högt i energi att det krävs kortvågig strålning (UV-B eller kortare) för att excitera nivåerna. Kortvågig strålning innebär ju stor energi enligt uttrycket

E = hv = hc/l

där E är strålningens energi, h och c konstanter och l är våglängden.

För mer om UV-ljus, speciellt om hur människan påverkas, se fråga 12865 .



/Peter E

Nyckelord: UV-ljus [13]; #ljus [63];

*

Ljud-Ljus-Vågor [17411]

Fråga:
Hej! Vi har optik nu och det är något som förbryllat mig varje gång jag ska förklara hur bilden skapas i ögat. I alla våra böcker står det att fokus ligger i gula fläcken och att bilden dör blir upp-och-ned-vänd. Hur kan den bli det då fokus ligger där? Är det så att fokus ligger precis framför ingröpningen i gula fläcken och bilden vänds innan?
/Catharina R, Sollentuna Musikklasser, Sollentuna

Svar:
Catharina! Låt oss börja med ett par definitioner:

Gula fläcken är ett område i näthinnan där synreceptorerna sitter som tätast. Det är den del av synfältet där man ser skarpast.

Fokus, brännpunkt eller fokalpunkt för en konvex (positiv) lins är den punkt där ljusstrålar konvergerar (går samman) när linsen riktas rakt mot en oändligt avlägsen ljuskälla. Ordet brännpunkt, liksom fokus (latinskt ord för härd), kommer av att den avlägsna ljuskällan Solen åstadkommer en brännande punkt just här.

Föremålet avbildas på näthinnan (av vilken gula fäcken är en del) av en enkel positiv lins. Bilden blir då upp-och-nervänd, se fråga 14748 .

Om föremålet ligger oändligt långt borta har man ett knippe med parallella strålar, se nedanstående figur. De blå strålarna kommer från en punkt på optiska axeln (N, linsens symmetriaxel) och bryts till fokus-punkten F'. Det svarta knippet kommer däremot från en punkt vid sidan av den optiska axeln, och bryts samman i en punkt i fokalplanet. Den fokuserade bilden för oändligt avstånd ligger alltså i ett plan som kallas fokalplan.

Om föremålet S i stället ligger närmre linsen, så kommer den fokuserade bilden att hamna utanför fokus i punkten F. I ett teleskop skulle man förskjuta okularet fram och tillbaka längs den optiska axeln tills man ser en skarp bild. För ögat är det emellertid lite annorlunda eftersom man inte kan ändra avståndet mellan linsen och näthinnan. I stället justeras linsens styrka så att det föremål man vill titta på är skarpt på näthinnan.

Jag tror att ditt problem är en alltför generell användning av begreppet fokus, se kommentaren i Fokus_(optik)#En_annan_betydelse_av_fokus . Fokus och fokalplan är fixa och väldefinierade för en lins med konstant styrka. Ett föremål närmare linsen "fokuseras" i ett plan man bör kalla bildplan, inte fokus eller fokalplan.

Se även fråga 7834 .



/Peter E

Nyckelord: ögat [18]; #ljus [63];

1 http://user.tninet.se/~bxf528q/eye/eyefakt2.html

*

Ljud-Ljus-Vågor [17168]

Fråga:
Hej! Jag funderar på begreppet reflektion av ljus och ljuskälla. Vi kan med ögat uppfatta föremål och dess "färger" (utgår här från solen som ljuskälla) pga att föremålen avger energi med olika våglängder. Detta gör de när elektoner exiteras med hjälp av en energikälla, i detta fall solljus, och varefter de sedan återgår till en lägre energinivå och stålningsenergi avges. Om föremålet som träffas av solljusets respektive energiinnehåll, och därmed ljus med olika våglängder, uppfattas som grönt innebär det att endast de atomer var elektroner fått rätt energimängd kan exiteras och avge stålning. Vi uppfattar föremålets färg som grön.Den övriga energimängden från ljuset absorberas av föremålet i form av värmeenergi. (förenklad version) Om all stålningsenergi absorberas uppfattas inga färger och föremålet sägs vara svart. Är det då korrekt att vi säger att förmål är gröna pga det gröna ljuset reflekteras? Resultatet vid spegelbilder är att ljus med olika vågländer lämnar föremålet och regelbundet reflekteras pga kollisioner mot elektroner i spegelmaterialet. Därefter når dessa våglängder, med sin nya riktning, vårt öga och vi uppfattar att kroppen utsänder detta ljus från en punkt bakom spegeln. Mina funderingar kan sammanfattas i följande frågor.

Kan det anses korrekt att vi uttrycker oss som att att förmål är gröna pga det gröna ljuset reflekteras?

Om det gröna föremålet hade reflekterat det gröna ljuset skulle det inte innebära att det är delar av solen vi skulle uppfatta utifrån spegelanalogin?

Varför betraktas inte "färgade" föremål som ljuskälla om de avger ljus på samma sätt som tex solen gör.
/Kjersti E, Varberg

Svar:
Kjersti! Det vi kallar för färg är dels beroende av ljuskällan och dels av de våglängdskänsliga receptorerna i ögat (tapparna). Se färg/färgseende för mer om detta.

Om vi begränsar oss till reflekterat vitt ljus (solljus) så är det korrekt att vi ser de färger som inte absorberas. Se t.ex. fråga 10888 om varför växter är gröna. Observera att det vi kallar reflektion (reflexion , Reflection_(physics) ) innebär att ljuset återkastas omedelbart och oförändrat. Om ett föremål ser vitt ut betyder det att alla färger reflekteras. Det sker alltså inte som t.ex. i fluorescens att atomerna exciteras och sedan sänder ut ljus. En typiskt reflekterande yta (metall) innehåller fria elektroner och det är dessa som orsakar reflektionen.

Ljuskälla är inget strikt definierat begrepp, men man kallar knappast en kropp som bara reflekterar inkommande ljus för en ljuskälla. De vanligaste ljuskällorna (List_of_light_sources ) är temperaturstrålare (temperaturstrålning ) eller luminicerande (luminiscens ).

Se även Diffuse_reflection .
/Peter E

Nyckelord: färg/färgseende [39]; ljusreflektion [18]; #ljus [63];

*

Ljud-Ljus-Vågor [16989]

Fråga:
Har fotonen massa?
/Veckans fråga

Ursprunglig fråga:
Enligt relativitetsteorin har fotonen ingen massa, pga. att den rör sig med ljushastigheten. Samtidigt påverkas fotoner av gravitationen (avböjning av ljus som passerar nära en massa, gravitationell rödförskjutning).

Jag har läst att man har visat experimentelt att ljuset ändrar sin frekvens(minsking av energi) när den färdas från jorden. Tycker att det låter konstigt!

Min lärare sa att "Fotonen har massan noll och det är därför den rör sig med ljusfarten. Alla objekt som har massa rör sig med farter som är mindre än ljusfarten." Hur kan man då förklara det med experimentet där fotonen kan påverkas av gravitationsfält??

Det är helt förvirrat för mig då jag läser att fotoner inte har någon massa men den kan ändå ha partikelegenskaper. Hur kan det komma sig? Och vad betyder egentligen dessa experiment som har vissat att fotoner påverkas av gravitationskraften och att den har rörelsemängd (compton spridning).
/Ali Z, borgarskolan, malmö

Svar:
Ali! Det var många svåra frågor. Låt oss börja med partikel-egenskaper. Jag tycker inte man skall föreställa sig en foton varken som en partikel eller en våg. En foton är en foton som lånat egenskaper både från partiklar och vågor.

Fotonens massa: Fotonen har energin E=hv. Eftersom energi och massa är ekvivalenta (E=mc2), så har fotonen massa. Man kan emellertid inte tala om fotonens vilomassa eftersom begreppet en stillastående foton saknar mening.

Experimentella bevis för att fotonen saknar vilomassa kommer bland annat från det faktum att den elektrostatiska kraften (Coulombs lag, som ju förklaras genom ett utbyte av virtuella fotoner) varierar som 1/r2 En utbytespartikel med ändlig vilomassa hade givit ett annat avståndsberoende.

Se fråga 16939 för mer om historien bakom fotonbegreppet och länk 1 för en mer detaljerad framställning. Wikipedia-artikeln Photon är mycket bra, medan den svenska versionen är OK men inte särskilt omfattande: Foton .

Fotoner påverkas på två sätt av gravitationsfält:

1 Avböjning, t.ex. vid passage nära solen. Detta har behandlats ganska detaljerat i fråga 16021 .

2 Gravitationell rödförskjutning.

Till skillnad från avböjning så kräver faktiskt en tillfredsställande behandling av gravitationell rödförskjutning bara enkel klassisk fysik och speciella relativitetsteorins E=mc2. Börja med att studera den enkla animeringen under länk 2!

Den potentiella energin hos elektronerna i övre läget är (mgh)

2me*g*H

Om vi förlänger med c2 får vi

2mec2*(g*H/c2) = Efoton*(g*H/c2)

Vi får det relativa skiftet för höjdskillnaden 22.5 m (ref. 1):

DEfoton/Efoton = gH/c2 = 9.803*22.5/(2.998*108)2 = 2.454 10-15

Det relativa skiftet har uppmätts (ref. 1) med hjälp av mössbauerspektroskopi (se nedan) för 14.4 keV fotoner från 57Co-sönderfall. Det uppmätta resultatet 2.451 10-15 (med c:a 1% osäkerhet) är i bra överensstämmelse med detta.

Observera att det enda antagande vi gör är energins bevarande, vilket är en av fysikens grundläggande och mest etablerade lagar.

Observera även att man kan se skiftet till större våglängd (och därmed lägre frekvens) när fotonerna går uppåt är ekvivalent med att klockan går långsammare i ett starkare gravitationsfält.

Liknande experiment med hjälp av satelliter som sänder ut en mycket välbestämd frekvens har bekräftat Einsteins teori med en noggranhet bättre än 1 del på 104.

Se vidare General_relativity , Gravitational_redshift och Pound-Rebka_experiment .

Se även http://fy.chalmers.se/~f1xjk/FysikaliskaPrinciper/FOREL.lp2/F16/F16.html

Mössbauerspektroskopi

Mössbauer-effekten är rekylfri emission och absorption av gammastrålning från atomkärnor. När en atomkärna utsänder ett gammakvantum förloras i en del av energin till kärnans rekyl liksom vid absorption i en absorberande kärna. Detta eftersom både energi och rörelsemängd måste bevaras i processen.

I mössbauereffekten elimineras förlusterna dels genom att de radioaktiva kärnorna sitter i en kristall som tar upp rekylen, så att emission och absorption kan ske vid samma energi och dels genom att man kan kompensera energiförlusten genom att låta den utsändande kärnan röra sig.

Eftersom rekylen är mycket liten räcker det med en mycket måttlig hastighet på några mm/sekund, se nedanstående bild där det lilla diagrammet är en plot av ett mössbauerspektrum med hastighet på den horisontella axeln och observerad intensitet på den vertikala. Dippen i spektrum reflekterar det exciterade tillståndets vidd DE. Vidden är relaterad till tillståndets livslängd enligt Heisenbergs obestämdhetsrelation :

DDt = h / 4p ~ 10-34 Js

Se vidare Mössbauer_spectroscopy och mössbauer-effekten . Se även fråga 14685 . ____________________________________________________________
1 Pound and Snider, Physical Review Letters Vol 13, 18 (1964) 539



/Peter E

Nyckelord: relativitetsteorin, allmänna [33]; #ljus [63];

1 http://www.desy.de/user/projects/Physics/ParticleAndNuclear/photon_mass.html
2 http://fragelada.fysik.org/reltest1/frame.htm

*

Ljud-Ljus-Vågor [16939]

Fråga:
Hur har man kommit fram till att fotonens energi är W=hf?
/Veckans fråga

Ursprunglig fråga:
Hej! hur har man kommit fram till att fotonens energi är W=hf, där h är planks konstant och f är frekvensen? finns det en svensk länk där man kan läsa om det i djupet tack!!
/Ali Z, malmö borgarskolan, malmö

Svar:
Hej Ali! Bra fråga! Formeln för fotonens energi

E = hv (av konvention använder man oftast v [grekiska ny] för fotonens frekvens)

är ju så djupt rotad i den moderna fysiken att man kanske glömmer vad den kommer ifrån.

När det gäller fysikaliska samband uppkommer de typiskt på ett av två sätt:

1 ett experimentellt uppmätt samband eller lag

2 ett antagande som leder till andra samband som kan verifieras experimentellt

eller en kombination av 1 och 2

När det gäller fotonens energi är det till att börja med fall 2: Max Planck (Max_Planck ) gjorde antagandet att energin var proportionell mot frevensen för att härleda ett fungerande uttryck för den den experimentellt observerade fördelningen hos temperaturstrålning (1900), se Plancks strålningslag , speciellt fråga 12397 och Planck's_law (den senare på engelska).

Bilden nedan från Wikimedia Commons Ultraviolet_catastrophe ) visar uppmätt temperaturstålning för tre olika temperaturer (nedre kurvorna). Den övre, svarta kurvan visar den klassiska förutsägelsen (Rayleigh–Jeans law). Som synes avviker den senare mycket från den observerade fördelningen, speciellt för korta våglängder.

Plancks uttryck representerade uppmätta data mycket bra även för korta våglängder. Plancks antagande att energin var given av strålningens frekvens var en avvikelse från den klassiska teorin där energin gavs av amplituden hos strålningen. Utan att veta det förebådade Planck den kommande kvantmekaniken.

Einstein var i sin artikel om fotoelektriska effekten (1905) mycket tydlig med kvantiseringen, och införde begreppet foton för en "ljus-partikel". I fråga 2931 visas data för fotoelektriska effekten som visar proportionaliteten mellan energi och frekvens.

Det mest direkta beviset kom genom Bohrs atommodell (1913). Man kunde bygga upp energidiagram där skillnaden i energin mellan två tillstånd var lika med energin hos fotonen som utsändes vid en övergång. Man kunde mäta våglängden och med hjälp av det generella sambandet mellan vågens utbredningshastighet c, våglängden l och frekvensen v

c = l*v

verifiera proportionaliteten mellan energi och frekvens. Senare infördes namnet Plancks konstant h för denna proportionalitetskonstant.

År 1923 verifierade Arthur Compton sambandet återigen genom sitt experiment att sprida fotoner på elektroner, se comptonspridning .

Länk 1 är en intressant artikel om Max Planck och länk 2 beskriver den historiska utvecklingen av atomteorin.

/*fa*



/Peter E

Nyckelord: Plancks strålningslag [6]; fysik [10]; elektromagnetisk strålning [21]; #ljus [63];

1 http://www.fof.se/tidning/2002/3/max-planck
2 http://www.pixe.lth.se/bossen/fysik/history1.htm

*

Ljud-Ljus-Vågor [16457]

Fråga:
Ljuset bryts ju när det går från luft till glas. Hur kommer det sig då att inte allting ser ändrat ut när man tittar ut genom fönstret?
/Veckans fråga

Ursprunglig fråga:
Ljuset bryts ju när det går från luft till glas. Hur kommer det sig då att inte allting ser ändrat ut när man tittar ut genom fönstret?
/maria z, göingeskolan, broby

Svar:
Maria! Mycket bra fråga! Den orsakade oss en hel del huvudbry ! Problemet är lite besläktat med kromatisk aberration (Chromatic_aberration ) vilket är ett bildfel hos en lins. Först kan man konstatera att du om du tittar ut vinkelrätt mot rutan, så händer inget. Ljuset går rakt igenom. Om du däremot tittar i en vinkel på säg 45o så finns det en liten effekt.

Eftersom glasytorna på fram och baksidan av rutan är parallella, så kommer en enfärgad stråle inte att ändra riktning: brytningen i en första ytan kompenseras exakt av en motsatt brytning i den andra ytan. Vad som alltså händer är att allt du ser genom rutan blir lite förskjutet i sidled, se nedanstående figur.

Om vi emellertid har olika färger, så är ju brytningsindex lite olika. Enligt figuren i Index_of_refraction#Dispersion_and_absorption är brytningsindex för den nedersta glassorten 1.49 för rött ljus och 1.50 för blått. Vi kan med hjälp av brytningslagen beräkna vinkeln a (infallsvinkeln i är 45o) för rött ljus och för blått ljus:

Blått: sin(45)/sin(a) = 1.50/1

dvs a = 28.13

Rött: sin(45)/sin(a) = 1.49/1

dvs a = 28.33

Förskjutningen x från ingång till utgång för en 5 mm tjock ruta ges av

x = 5*tan(a)

Vi får alltså för blått ljus

x = 5*tan(28.13) = 2.673 mm

På samma sätt får vi för rött ljus x = 2.695 mm

Skillnaden i utträdespunkt blir alltså 2.695 - 2.673 = 0.022 mm.

Avståndet mellan den blå och den röda strålen blir då 0.022*sin(45) = 0.016 mm = 16 mikrometer.

Enligt länk 1 är det typiska avståndet mellan tapparna i ögat c:a 10 mikrometer. Effekten är alltså mycket liten men bör kunna observeras under gynnsamma förhållanden. Vad man behöver är en liten och stark vit ljuskälla. Om man betraktar denna med mycket snett infall bör man kunna se ett litet spektrum med rött i ena änden och blått i den andra. Men effekten är så liten att man inte märker den under normala förhållanden.



/Peter E

Nyckelord: ljusbrytning [26]; #ljus [63];

1 http://www.cis.rit.edu/people/faculty/montag/vandplite/pages/chap_9/ch9p1.html

*

Ljud-Ljus-Vågor [16165]

Fråga:
Kan LED-lampor vara skadliga?
/Veckans fråga

Ursprunglig fråga:
Inverkan av monokromatiskt ljus på människan.

Hej, med anledning av kommande förbud mot kvicksilverlampor, och allmän energibesparing, är det tänkt att till stor del ersätta dessa med LED-lampor.

1. Vilket spektra har det monokromatiska vita ljus som utsänds av LED? Vad är detta för en paradox, då vitt ljus består av "alla" frekvenser och monokromatiskt ljus defineras av singel-frekves-strålning?

2. Finns det någon forskning på monokromatisk ljusets inverkan på mäniskan/organismer under lång tid, även då pulserande monokromatiskt ljus (LED-dimmer använder frekvenser mellan 10KHz-30KHz)? Jag tänker här närmast på påverkan på ögats/hjärnans reseptorer och produktion av Melatomin, Kortisol och andra ämnen ( även Ljus & Färg terapi - undantaget behandling av hudåkommor med monokromatiskt pulserande ljus av hög effekt, förutom möjligtvis forskning på skyddsutrustning för dessa).

3.Vad vet vi om effekterna av (super-)högeffektiva LED och dess påverkan på ögat (jämfört med Laser)? Jag tänker närmast på skadligheten av att titta rätt på ljuskällan, om den inte är försedd med skyddsglas eller på annat sätt reflekteras - finns det någon som helst information eller rekomendation/reglering innom EU idag?

Tacksam för något att nysta vidare på...
/Manfred B, Mölndal

Svar:
1 Det finns i princip två sätt att åstadkomma vitt ljus med en lysdiod (LED).

a Antingen har man en LED för varje grundfärg (röd, grön, blå), se nedan. Om man blandar ljuset från dessa får man vad som av ögat uppfattas som vitt ljus (se färg/färgseende ). Detta är samma teknik som man använder sig av i en TV. Spektrum är en topp vid vardera rött, grönt och blått, alltså ingalunda kontinuerligt.

b Eller så har man en blå (eller UV) LED och fluorescerande material. Man får då en kontinuerlig våglängdsfördelning som visas i figuren i fråga 12571 . Våglängdsfördelningen är ingalunda den normala från temperaturstrålning (solen, glödlampor), se fråga 12564 .

2 Jag tror inte det finns någon forskning om inverkan från LED-ljus. Typ a bör knappast vara farlig, och så höga frekvenser uppfattas inte av ögat (för en bildskärm anses 100 Hz ge stadigt ljus). Men är det skadligt? Det vet man inte säkert. Typ b kan vara skadliga om alltför mycket av UV-ljuset kommer ut.

3 Vet jag i varje fall ingenting om. När LED blir vanligare kommer det säkert fram larm som det gjort om farligheten hos bildskärmar, mobiltelefoner, fält från kraftledningar mm. Jag har en känsla av att vi börjar använda nya uppfinningar, och i en del fall när de visar sig skadliga (t.ex. röntgen och radioaktivitet) så inför vi restriktioner i efterhand. Om varje tillverkare absolut säkert skulle kunna bevisa att hans produkt är säker, så skulle vi inte få några nya produkter. Men farlig som en stark laser med koherent ljus (samlat även på stora avstånd) är den säkert inte. Man skall ju kunna använda den för belysning och displayer!

Se vidare bra artiklar med fler länkar i Wikipedia: Light-emitting_diode och LED_lamp . Lysdiod är på svenska, men inte lika bra.



/Peter E

Nyckelord: lysdiod [14]; lågenergilampa [13]; #ljus [63];

1 http://www.etn.se/index.php?option=com_content&view=article&id=57126&via=r

*

Ljud-Ljus-Vågor [16135]

Fråga:
Varför har ljuset så många olika färger?
/Veckans fråga

Ursprunglig fråga:
Varför har ljuset så många olika färger?
/Allan E, alviksskolan, bromma

Svar:
Allan! Först måste vi fundera på vad är färg? Det är en benämning vi hittat på för synintryck från ljus av olika våglängder. Färg har alltså att göra med hur ögat skiljer på olika våglängder. I det mänskliga ögat finns det tre typer av färgkänsliga receptorer, s.k. tappar. Känsligheten för dessa framgår av figuren i fråga 13824 nedan. Vi har alltså en typ av tappar som i huvudsak är känslig för rött (som vi kallar r), en som är känslig för grönt (g) och en för blått (b). Det är då balansen mellan stimuleringen av r, g och b som avgör vilken färg vi uppfattar.

Det är ingen tillfällighet att tapparna är känsliga för just dessa våglängder: de ligger omkring maximum intensitet i solens spektrum och de absorberas inte av atmosfären. Tappar känsliga för helt andra våglängder skulle vara utan värde, så de skulle inte ha utvecklats.

Med tre olika sortes receptorer finns det många möjliga kombinationer, så därför finns många färger. Sedan är det ganska godtyckligt hur många färger vi givit namn (mörkvitt förekommer t.ex. bara i Bengt Grives konståkningsreferat). Men om vi bara haft två typer av tappar, så hade vi uppfattat färre färger, se Color_blindness och bilden nedan från Wikimedia Commons. I är hur en person med normalt färgseende uppfattar den amerikanska flaggan. Om r-tapparna fattas ser flaggan ut som II, dvs utan den röda färgen. Om man bara har en sorts fungerande tappar uppfattar man bara en gråskala som flaggan V.

Se vidare färg/färgseende , länk 1 nedan, Färgseende och Color_vision . I fråga 5381 finns lite om färgblindhet.



/Peter E

Se även fråga 13824 och fråga 5381

Nyckelord: färg/färgseende [39]; #ljus [63];

1 http://www.moderskeppet.se/grundlaggande_farglara.asp

*

Ljud-Ljus-Vågor [15928]

Fråga:
Varför gnistrar diamantringar vackrare i sol-, stearin-, och halogenljus än i vanlig glödlampe- eller lysrörsljus.
/Veckans fråga

Ursprunglig fråga:
Jag fick en fråga av en elev om varför diamantringar gnistrar vackrare i sol-, stearin-, och halogenljus än i vanlig glödlampe- eller lysrörsljus. Har det något med våglängder att göra?
/Louise N, Klöverbackens skola, Kungälv

Svar:
Louise! Skönhet ligger i betraktarens öga . Jag tror inte det har så mycket att göra med vilka våglängder ljuskällan sänder ut - för ögat är det inte så stor skillnad på de ljuskällor du nämner. Jag tror att det är viktigare att ljuskällan är liten (nära punktformig) och inte diffus (utbredd). Om du t.ex. har ett stearinljus som källa får du en massa spegelbilder av ljuslågan i diamanten. Det ser mycket vackrare ut är spegelbilden av t.ex. ett lysrör.

En del av ljuset kommer även till ögat efter att ha passerat genom delar av diamanten och därmed delats upp i olika färger som ett spektrum. Om ljuskällan är stor kommer spektrum att bli otydligt (de olika färgerna överlappar och ger intryck av vitt ljus). Det är därför man har en smal ingångsspalt i en spektrograf.

Se även fråga 14331 .
/Peter E

Nyckelord: *vardagsfysik [64]; #ljus [63];

*

Ljud-Ljus-Vågor [15785]

Fråga:
Hej. Vi pratade om ljus på NO lektionen, och då förklarade läraren varför polariserade solglasögon var så bra att använda.

Min fråga är: Man kan ju lägga polariserat glas så det knappt släpper genom något ljus alls (så att dom ligger på 2 olika håll). Släpper de då igenom IR-ljus? Min lärare visste inte detta och ja skulle vilja veta om de gör det.
/Fredrik J, Rolfstorp, Roflstorp

Svar:
Fredrik! Ett polaroidfilter är bara effektivt för ett begränsat vådlängdsområde. Det betyder att korsade polaroidfilter avsedda för synligt ljus fungerar dåligt för IR. Sedan är det UV du behöver skydda ögonen för och det gör solglasögon med rätt sorts plast/glas.

Att använda korsade polaroider för att skydda ögonen är naturligtvis en effektiv men inte särskilt användbar metod. Eftersom inget ljus släpps igenom kan du lika gärna använda en svart bindel!

För en gång skull är det aktuella modet med stora böjda solglasögon som täcker även sidorna en bra idé. De skyddar (om de är gjorda av UV-absorberande material) ögonen även från strålning som kommer från sidan. Så förutom att man ser cool ut visar man att man är klok .

Se vidare nedanstående länk om polaroidglasögon.
/Peter E

Nyckelord: polaroidglasögon [3]; #ljus [63];

*

Blandat, Ljud-Ljus-Vågor [15609]

Fråga:
Hur kan jag förklara för mina sjuor vad elektromagnetisk strålning är?
/Veckans fråga

Ursprunglig fråga:
Hej, hur kan jag förklara för mina sjuor vad elektromagnetisk strålning är? Hur jag än försöker föklara så verkar inte de förstå. Är mycket, mycket tacksam för hjälp !
/Jonas H

Svar:
Hej Jonas! Ja, det är inte lätt! Försök med Maxwells ekvationer , dom säger allt om elektromagnetisk strålning. Skämt åsodo , detta är ett problem. Vi måste acceptera två fakta vad gäller fysik och även annan naturvetenskap:

* Djupare förståelse för många fenomen kräver goda kunskaper i matematik och fysik.

* På en fundamental nivå förstår vi ingenting. Ingen vet t.ex. varför två massor attraherar varandra, varför elektromagnetisk strålning finns eller varför elektronens massa är vad den är.

När det gäller att undervisa i fysik tycker jag att man skall följa nedanstående punkter: alltså börja med naturvetenskapens grundvalar experiment och observationer och inte alltför tidigt teoretisera för mycket. Fysikundervisning utartar alltför ofta i att lösa tillrättalagda problem genom att sätta in värden i en formel. Problemet reduceras då ofta till ett meningslöst gissande vilken formel man skall använda. Det är bättre att koncentrera sig på experiment, och om man skall räkna så skall det vara verklighetsförankrade s.k. kontextrika problem - dvs sådana man ställs inför i verkligheten, se länk 1.

Glöm inte att fysik är en experimentell vetenskap! Albert Einstein (som för de flesta är urtypen av en teoretisk fysiker) har sagt:

"In the matter of physics, the first lessons should contain nothing but what is experimental and interesting to see. A pretty experiment is in itself often more valuable than twenty formulae extracted from our minds; it is particularly important that a young mind that has yet to find its way about in the world of phenomena should be spared from formulae altogether. In his physics they play exactly the same weird and fearful part as the figures of dates in Universal History."

Den franske matematikern Henri Poincaré hävdade att arbetet med fysik var som att driva ett bibliotek:

"Den experimentella fysiken svarar för förvärven, och det är bara den ensam som kan berika biblioteket. Den matematiska fysiken ska ordna katalogen. Biblioteket blir inte rikare om katalogen är lätt att hitta i, men läsaren kan utnyttja dess rikedomar bättre. Och genom att visa bibliotekarien på luckorna i samlingarna, kan resurserna användas klokt, vilket är helt avgörande, då resurserna alltid är bristfälliga."

För elektromagnetisk strålning är en möjlig utgångspunkt att beskriva användningen av och faran med strålning av olika våglängder, se fråga 15570 . Fråga 13590 ger en starkt förenklad model av vad elektromagnetisk strålning är. Vad gäller experiment med elektromagnetisk strålning är det lättast att begränsa sig till de typer vi kan uppfatta med sinnerna, dvs ljus och värmestrålning.

Nu tillbaka till lite allmänt om fysikens väsen:

1 Experimentera, observera
Detta är grunden för all naturvetenskap. Observationer i modern mening gjordes först av dansken Tycho Brahe (se Tycho_Brahe )under slutet av 1500-talet. Han mätte framför allt planeten Mars' bana. Italienaren Galileo Galilei (se Galileo_Galilei ) fortsatte 1609 observationerna av stjärnhimlen med det nyuppfunna teleskopet, men han utförde även många andra experiment framför allt i mekanik.

2 Se släktskap mellan olika fysikaliska fenomen
För många grenar av fysiken, som kan tyckas mycket olika, visar det sig att tolkningen av fenomen ofta har stora likheter. Teoretiska modeller från ett område kan ofta överföras med mindre modifikationer till ett annat. Alla mikroskopiska system (storlek atom och nedåt) styrs i princip av en enda ekvation, schrödingerekvationen .

3 Enkla modeller
För att föreställa sig ett fysikaliskt fenomen använder man sig ofta av enkla modeller. Bohr-atomen, med elektroner som rör sig i banor kring atomkärnan, beskriver en liten del av verkligheten och har därför ett visst värde. Man får emellertid inte övertolka förenklade modeller - modellen är inte verkligheten även om vissa modeller beskriver mätbara värden mycket väl.

4 Begränsa antalet grundläggande lagar
Olika fysikaliska lagar och teorier har olika dignitet eller status. Keplers lagar för planeternas rörelse kan t.ex. härledas från Newtons gravitationslag . Denna senare får därmed en högre status. På samma sätt beskriver Maxwells ekvationer (se ovan) det mesta som har att göra med elektricitet och magnetism, inklusive elektromagnetisk strålning. På så sätt kan vi beskriva världen med ett rimligt antal fundamentala lagar.

Följden Nicolaus Copernicus (idén att jorden kretsade kring solen), Tycho Brahe (noggranna observationer av planeternas rörelser), Galileo Galilei (mekanikförsök och observationer av astronomiska objekt med teleskop), Johannes Kepler (lagar för planetrörelsen) och Isaac Newton (universella gravitationslagen och Newtons rörelselagar) brukar framställas som början till ett modernt arbetssätt i vetenskapen. Detta brukar kallas den vetenskapliga revolutionen (Scientific Revolution, History_of_physics#Scientific_Revolution ).

Om man följer ovanstående punkter och bara går så långt som elevernas begreppsbild och andra färdigheter tillåter uppkommer inte problemet att läraren egentligen känner att han/hon inte förstår fenomenet fullständigt. Läraren får trösta sig med att det gör ingen!

Några nyckelord i frågelådan som är relevanta för naturvetenskaplig metod:
 * vetenskaplig metod
 * fysik
 * fysikalisk modell
 * fysik, förståelse av
 * fysik, nytta med
 * pseudovetenskap
 * parapsykologi
 * astrologi
/Peter E

Nyckelord: elektromagnetisk strålning [21]; fysik, förståelse av [17]; matematik i fysik [6]; #ljus [63];

1 http://groups.physics.umn.edu/physed/Research/CRP/crintro.html

*

Ljud-Ljus-Vågor [15556]

Fråga:
Hejsan Undrar hur det kommer sig att man på vissa glas/fönster kan se igenom det från ena hållet men inte från andra? Marcus
/marcus e, alléskolan, hallsberg

Svar:
Marcus! Som framgår av länk 1 och One-way_mirror så fungerar det inte så. Glasytan har ett tunnt skikt av aluminium så att hälften av det infallande ljuset reflekteras och hälften transmitteras. Detta gäller oavsett i vilken riktning ljuset går.

Effekten man har t.ex. i en identifikationsparad är att det ena rummet är väl upplyst och det andra är mörkt. Sett från det ljusa rummet dominerar reflexerna från fönstret så det fungerar som en spegel. Sett från det mörka rummet finns mycket lite reflexer, så man ser in i det ljusa rummet.

Addendum 16/1/08:

Marcus var inte riktigt nöjd med svaret, här är ett sammandrag av hans synpunkter:

Du verkar inte tro att det finns sådana fönster? Vi har ett sådant på vår skola. De är INTE symmetriska vad det gäller ljusgenomsläpplighet.
Marcus hade dessutom vänligheten att skicka mig en bit av foliet som sitter på fönstret.

Foliet ser uppenbarligen olika ut från olika håll: ena sidan är blank som en spegel medan den andra är mörkt matt. Detta förstärker uppenbarligen den avsedda effekten. Om man emellertid tittar genom foliet på t.ex. en lampa, så ser man tydligt att transmissionen är samma från båda hållen. Det måste vara så - om man kunde konstruera en perfekt "diod" för ljus skulle våra atomfysiker bli extatiska. I Wikipedia-artikeln Mirror#Two-way_versus_one-way_mirrors_and_windows står det:

A true one-way mirror that actually allows light to be transmitted in one direction only without requiring external energy is not possible as it violates the second law of thermodynamics.
(Second_law_of_thermodynamics )

Med en envägsspegel skulle man alltså utan att tillföra energi få värme att gå från en kallare till en varmare kropp i strid med termodynamikens andra huvudsats.

Det vi har är alltså ett halvgenomskinligt folie som är blankt på ena sidan och matt på den andra. Den blanka sidan ger maximalt störande reflexer (monteras alltså mot det man vill observera), medan reflexerna minimeras på den andra sidan för att inte störa observationen. Den matta sidan är antingen absorberande eller diffust spridande.

Jag vet inte hur man tillverkar foliet. I mikroskop ser man bara mörkare områden bland de genomskinliga (folien belystes i transmission). Jag gissar man förångar en metall som får fastna på en plastfolie. Den fria ytan skulle sedan kanske göras matt genom t.ex. oxidation. Men detta är bara en gissning.
/Peter E

Nyckelord: spegel [10]; genomskinlighet [18]; ljusreflektion [18]; #ljus [63]; termodynamik [17];

1 http://science.howstuffworks.com/question421.htm

*

Ljud-Ljus-Vågor [15480]

Fråga:
Hej! Det verkar som om andelen reflekterat ljus när man speglas i en glasruta ökar med ökande infallsvinkel. Stämmer iakttagelsen och följer andelen någon enkel(?) formel? (En möjlighet annars är ju att vid infallsvinkel 0 grader så skulle utifrån kommande ljus spela större roll och "dränka det reflekterade mera effektivt" så att det tycks relativt svagare än det är och snett infallande, utifrån kommande ljus kanske försvagas i glasrutan p g a längre väg i glas.)
/Thomas Å, Arlandagymnasiet, Märsta

Svar:
För vinkelrätt infall mot en gränsyta mellan brytningsindex 1 (vakuum) och n är reflektionsförmågan R

R = (n-1)2/(n+1)2

För glas med n=1.5 får vi

R = (0.5)2/(2.5)2 =(1/5)2 = 0.04 = 4%

För snett infall ökar reflektionen med ökande infallsvinkel (mäts i förhållande till normalen på gränsytan) om vi kan bortse från polarisationen. Uttrycket är inte allför komplicerat, se nedanstående figur från Fresnel_equations . Figuren visar reflektionskoefficienten som funktion av infallsvinkeln dels för ljus från ett tunnare till ett tätare medium (n1 < n2) och från ett tätare till ett tunnare medium (n1 > n2). I det senare fallet får man totalreflektion från en viss gränsvinkel. De båda kurvorna i varje diagram är infallande strålens polarisation.



/Peter E

Nyckelord: ljusreflektion [18]; totalreflektion [9]; #ljus [63];

*

Ljud-Ljus-Vågor [15445]

Fråga:
Hej! Jag undrar följande: När två ljusstrålar interfererar, skapas ofta ett interferensmönster. Då kan man tänka sig att energin bevaras genom att intensiteten blir större på vissa ställen och mindre på andra. Men om man tar två lasrar och placerar den ena en halv våglängd bakom den andra, så att total utsläckning sker, vad händer då med energin?
/Alexander B, Kemistudent, Göteborg

Svar:
Alexander! Nej, det är inget problem med bevarandet av energin. Förklaringen är densamma som i fråga 965 att intensiteten hamnar någon annanstans! För att sammanföra dina två laserstrålar så att de går i exakt samma riktning måste du använda dig av halvgenomskinliga speglar. Detta betyder att om strålarna i en riktning interfererar destruktivt, så kommer dom att interferera konstruktivt i en annan riktning. Det går inte att lura energiprincipen.

Det är inte helt lätt att få en korrekt bild av ljus och interferensfenomen. Man säger att ljuset i vissa sammanhang uppvisar vågegenskaper och i andra partikelegenskaper. Det är viktigt att inte lägga för mycket i dessa tolkningar - ljuset är varken en ren våg eller en ren partikel. Ljuset är helt enkelt ljus.

Ett annat välkänt problem är att förstå att man får interferens även när ljusstyrkan är så låg att vi bara har att göra med en enda foton. Det visar sig att fotonen kan interferera med sig själv.

I länk 1 finns en korrekt förklaring:

The correct answer is "when two waves destructively interfere, the energy goes to the places of constructive interference." This statement, however, can be misleading. It's not like energy travels to a point in space of destructive interference, gets sucked into a vortex, gets destroyed, and magically appears in locations of constructive interference. The problem with this vortex picture is that "energy" is portrayed as little balls darting about. More correctly, interference is a purely wave phenomenon (when quantum particles interfere, it's only by acting like waves). Energy is carried by a spread out, connected thing we call a wave. When we say that two waves destructively interfere, we do not mean that energy is literally destroyed at these points. We mean that energy never really reaches these points. The energy goes directly to the points of constructive interference. Again, it's not like energy in this context is a ball that gets magically rerouted to the correct points of constructive interference. Rather, the waves spread out through all space, and simply have no energy at the points of destructive interference.

For instance, consider thin film interference. If you shine a monochromatic wave of the right frequency at a thin film, the wave reflected off the front of the film and the wave reflected off the back of the film destructively interfere at all points in space. As a result, there is zero energy reflected from a thin film a the right frequency. All of the energy is transmitted. But it's not like the energy gets reflected off the thin film, notices that it has entered a region of destructive interference, and quickly sneaks back into the thin film. Rather, the energy never gets reflected in the first place. The statement "the wave reflected off the front of the film and the wave reflected off the back of the film destructively interfere" is a purely mathematical statement and not a physical one. In the physical world, there are no independently-existent wave components; there is just a single electromagnetic field that fluctuates in a complicated way that can modelled as the sum of sine waves. Wave components are mathematical entities humans use to make the math easier. Physically, what happens is that the fields and material in the thin film interact such that a reflected wave is never created in the first place.

Se även länk 2 nedan.
/Peter E

Nyckelord: interferens [14]; #ljus [63];

1 https://www.reddit.com/r/askscience/comments/1z6crk/when_two_waves_destructively_interfere_where_does/
2 http://www.rp-photonics.com/spotlight_2007_09_27.html

*

Ljud-Ljus-Vågor [15260]

Fråga:
Jag undrar om ni vet hur långt ljus kan gå i vatten. Jag arbetar nämligen med livsformer under isen på antarktis och undrar om det finns någon möjlighet att någon form av ljus eller strålning (något energrikt) kan gå igenom flera tusen meter vatten. Om det kan gå igenom vatten på sådant avstånd,skulle det då kunna ske på samma sätt genom is?

Är den energin tillräcklig för att mikrober skall kunna ha den som energikälla?
/Sebastian Å, Sjölins Gymnasium, Göteborg

Svar:
Det beror mycket på hur klart vattnet är men för det klaraste havsvattnet återstår mindre än 0.5% av ljuset efter 200 m, se länk 1. Det är inte någon stor skillnad på vatten och ren, klar is. Is (speciellt is bildad av komprimerad snö) innehåller emellertid ofta luftbubblor, och då är absorptionen mycket högre.

Vostok-sjön (se länk 2) är en stor insjö 4 km under isen i Antarktis. Det är uppenbart att nästan inget ljus når dit. Man har emellertid borrat nästan ända ner till sjön och hittat bakterier.

Allt liv behöver energi (se livets uppkomst för kriterier för liv), men inte nödvändigtvis ljus. Växter får energi från ljus och bygger upp kolhydrater. Djur kräver inte ljus för sin energiförsörjning (men säkert för andra ändamål) utan får energi direkt eller indirekt från de av växterna tillverkade kolhydraterna.

Det finns livsformer i djuphavet vid utlopp av vamvatten från vulkanism (extremofiler, googla black smokers). Några organismer använder det upp mot 400 grader varma vattnet som energikälla. Andra organismer använder de av de första bildade kemiska föreningarna som energikälla.

Vostok-sjön är mycket intressant eftersom förhållanden mycket liknar de på jupitermånen Europa. Om det finns något liv i Vostok-sjön beror framför allt på om där finns någon energikälla. Vi får se vad man hittar när man hittat en säker metod att studera sjön med.
/Peter E

Nyckelord: astrobiologi [9]; *biologi [20]; #ljus [63];

1 http://oceanworld.tamu.edu/resources/ocng_textbook/chapter06/chapter06_10.htm
2 http://en.wikipedia.org/wiki/Lake_Vostok

*

Ljud-Ljus-Vågor [15042]

Fråga:
Varför syns inte alla linjer i emissionsspektrum i ett absorptionsspektrum?
/Veckans fråga

Ursprunglig fråga:
När ljus passerar genom vätgas vid rumstemperatur kan absorptionslinjer som motsvarar Lymanserien observeras? Varför syns inte de andra serierna? Vad spelar rumstemperaturen för roll? Borde det inte vara de andra serierna som syns, med tanke på att Lymanserien omfattar energisprång ner till grundtillståndet, vilket ger ultraviolett ljus som inte är synligt?
/Petra L, Södra Latin, Stockholm

Svar:
En absorptionslinje uppkommer när en foton träffar en atom och lyfter upp (exiterar) en elektron till ett högre liggande tillstånd. En emissionslinje uppkommer när elektronen hoppar tillbaka till ett lägre tillstånd. Se fråga 176 för en förklaring hur emissions- och absorptionsspekta uppkommer. Vad som inte framkommer där är att inte alla emissionslinjer förekommer i absorptionsspektrum. Om den absorberande gasen är kall, så ser man bara övergångar från grundtillståndet, a och b i nedanstående bild. Detta för att alla atomer befinner sig i grundtillståndet. I emissionsspektrum, däremot, ser man alla tre övergångarna a, b och c.

Lymanserien är den serie i vätespektrum som slutar i grundtillståndet, och eftersom alla atomer vid rumstemperatur befinner sig i grundtillståndet ser man bara lymanserien i absorption. Om temperaturen är betydligt högre kan en del av atomerna tillfälligtvis befinna sig i tillstånd ovanför grundtillståndet. Man kan då (som i solens spektrum) se även andra serier, t.ex. balmerserien, i absorption, se fråga 17723 .



/Peter E

Se även fråga 176

Nyckelord: spektrum [11]; #ljus [63];

*

Ljud-Ljus-Vågor [14936]

Fråga:
Hur fungerar selektiva absorbenter i solfångare?
/Veckans fråga

Ursprunglig fråga:
I termiska solfångare finns s.k. selektiva absorbenter. Dessa sätter de vanliga formlerna för svartkroppsstrålning och för absorbtion och reflektion ur spel. Hur har man lyckats med denna bedrift. Kan man skapa en sådan yta med de resurser som finns i en ordinär gymnasiefysiksal? Om möjligt önskas ett utförligt svar eller hänvisning till litteraturen.
/Anders K, Sollefteå

Svar:
Nej, selektiva absorbenter sätter inte några naturlagar ur spel! Selektiva absorbenter innebär att ytskiktet på solfångaren behandlats med ett ytskikt som skall ge hög absorption av solstrålning och låg emittans av värmestrålning.

Kirchhoffs strålningslag säger att absorpionsförmågan är proportionell mot emissionsförmågan vid en viss våglängd. Solens yttemperatur är c:a 6000 grader, och den mesta energin i solstrålningen ligger i synligt ljus 400-700 nm. Det är alltså i detta område man vill ha maximal absorptionsförmåga hos en solfångare. Normalt innehåller en solfångare vatten som värmebärare, så temperaturen är maximalt 100 grader. Vid denna temperatur ligger maximum hos temperaturstrålningen vid mycket längre våglängder - i infrarött (se fråga 12793).

Med Blackbody Radiation Applet kan man uppskatta maximum i energifördelningen för olika temperaturer. För 6000 K ligger maximum vid 500 nm och vid 350 K (c:a 80oC) vid 8000 nm.

Även naturen utnyttjar denna selektiva absorption i växthuseffekten . Solljuset går obehindrat igenom atmosfären och värmer upp jordytan. Värmestrålningen från jordytan hindras att försvinna ut i rymden av växthusgaser - framför allt vattenånga och koldioxid. Utan denna värmande effekt skulle jorden vara c:a 35 grader kallare i medeltemperatur än vad den är.

Se vidare länk 1, solenergi , temperaturstrålning och Plancks strålningslag .
/Peter E

Se även fråga 12793

Nyckelord: solenergi [14]; Kirchhoffs strålningslag [4]; temperaturstrålning [29]; strålning, in-/ut- [6]; #ljus [63];

1 http://www.iva.se/upload/Verksamhet/Projekt/Energiframsyn/El%20och%20V%C3%A4rme%20komplett3.pdf

*

Ljud-Ljus-Vågor [14908]

Fråga:
Hej! Det cirkulerar en fråga här hos oss. Om man hoppar ner i en 50 meter djup brunn och tittar upp på himlen en vacker klarblå sommardag, vad har då himlen för färg genom det lilla hål som blir i toppen av brunnen? Här sägs det att himlen ser ut som om det vore natt - svart med stjärnor på...
/Nina T, Komvux, Borås

Svar:
Hej Nina i Textilstaden! Nej, detta är en fabel! Den lilla fläcken du ser är lika klarblå som när du står på marken! Den är antagligen i stället ljusare eftersom mörkret i brunnen gör att ögonen mörkeradapteras.

Varför uppkommer en sådan här missuppfattning? Kanske resonerar man som så att den blå färgen uppkommer genom att ljus från solen sprids av luften, och luften i brunnen belyses inte av solen. Problemet med detta är att det ändå finns flera mil atmosfär ovanför jordytan, så himlen blir blå ändå. Man skulle behöva stå inne i en flera mil hög skorsten. Då skulle himlen bli svart och man skulle kunna se stjärnor.

Kanske tror man att det fungerar på samma sätt som när man tittar genom ett teleskop med hög förstoring. Den blå himlen har en viss ljusstyrka per ytenhet. Teleskopet förstorar ytan, så ljusstyrkan per ytenhet avtar. En punktkälla som en stjärna blir däremot inte större, så kontrasten stjärna/himlen blir större. Man kan därför se åtminstone ljusstarka stjärnor i ett teleskop även på dagen.
/Peter E

Nyckelord: blå himmel [12]; #ljus [63];

*

Ljud-Ljus-Vågor [14613]

Fråga:
Jag håller på och utbildar mig till smed, och smider just nu eldstål. En fråga som jag ställt mig under arbetet med eldstålen är vilken metall som ger varmast gnistor. När jag slipar på stål i bandslipen så ger olika stål olikfärgade gnistor och jag har antagit att ju ljusare gnistor - desto varmare gnistor. Problemet är att olika legeringar ger olika färg på gnistorna och jag är osäker om mitt antagande ovan stämmer. Frågan till er är om det verkligen är så att färgen på gnistorna helt enkelt ger ett rättvist svar på temperaturen?
/Magnus M

Svar:
Magnus! Du har rätt i din misstanke att det inte är temperaturen som bestämmer färgen hos gnistorna. Olika legeringar innehåller olika ämnen. När atomerna förångas under hög temperatur kommer de att exciteras till högre tillstånd genom kollisioner. Dessa exciterade tillstånd sönderfaller sedan tillbaka grundtillståndet genom att sända ut ljus med en för atomslaget karakteristisk våglängd.

Detta används för att få vackra färger på fyrverkerier: natriumsalter ger gul färg, koppar blå, strontium röd och barium grön. Se vidare Fyrverkeri#Ingredienser_i_fyrverkerier och länk 1.

Ett temperaturstrålande gult objekt skulle ha en temperatur på c:a 6000 grader (som solens yta). Detta är uppenbarligen mycket högre temperatur än vad man har i en gnista!
/Peter E

Se även fråga 12409

Nyckelord: spektrum [11]; temperaturstrålning [29]; #ljus [63];

1 http://webmineral.com/help/FlameTest.shtml#.VNndQpV0zIU

*

Universum-Solen-Planeterna [13917]

Fråga:
Varför har vi årstider?
/Veckans fråga

Ursprunglig fråga:
Alla planeter som har en lutning har årstider. Men borde inte själva årstiderna bero på att ett område får olika mängd energi ifrån solen beroende på vart i året dom är. Och jag undrar också om det är möjligt att räkna ut hur mycket energy en plats får som är X° från ekvatorn, och planeten lutar Y°. är det möjligt?
/zelos j, rudbeck, örebro

Svar:
Jo, primärt beror årstiderna på att solenergin sprids ut på olika stora ytor beroende på vinkeln mellan jordytans plan och riktningen till solen, se figuren nedan. Vi kan även uttrycka det så att instrålningen (effekten hos inkommande strålning) beror på hur högt solen står på himlen. Dessutom påverkas naturligtvis instrålningen av att dagens längd varierar. En ytterligare effekt som påverkar temperaturen är jordytans albedo (reflektionsförmåga). Is/snö har högt albedo och tederar alltså att sänka temperaturen.

Eftersom man har utbyte av värme mellan olika delar av jorden (havsströmmar och vindar) blir det i själva verket mycket mer komplicerat att beräkna temperaturen vid en viss breddgrad för en given tid på året.

Om man placerar en yta på en kvadratmeter vinkelrätt mot solen utanför jordatmosfären kommer ytan att motta effekten 1370 W. Detta är vad man kallar solarkonstanten (1370 W/m2). Om vinkeln mellan ytan och riktningen till solen är i, så kommer effekten per kvadratmeter bli 1370*sini, dvs uppvärmningen blir mindre ju mindre i blir.

Tillägg om solarkonstanten

Man kan beräkna solarkonstanten från solens utvecklade effekt (luminositet) P (för detta värde och jordbanans radie R se Planetary Fact Sheets ):

solarkonstanten = P/(4pR2)

Uttrycket i nämnaren är ytan av ett klot med jordbanans radie. Med insatta värden får vi

solarkonstanten = 384.6*1024/(4p*(149.6*109)2) = 1368 W/m2.

Anmärkning: I själva verket har man bestämt solarkonstanten och från denna räknat ut solens utvecklade effekt.

Se fråga 16846 hur man uppskattar jordens medeltemperatur från solarkonstanten. I länk 1 uppskattas solens utstrålade effekt med hjälp av Stefan Boltzmans lag.

Se vidare solarkonstanten och Solar_constant .



/Peter E

Nyckelord: årstider [4]; solarkonstanten [6]; solenergi [14]; #ljus [63]; solens energiproduktion [9];

1 http://www-vaxten.slu.se/amnesingang/Naturvet/ovningar/solarkonst.htm

*

Elektricitet-Magnetism [13822]

Fråga:
Jag har läst här på frågesidan om magnetiska monopoler. Svaren ni ger innebär att inga teoretiska hinder finns för deras existens. Detta är fullständigt nonsens enligt min mening. Självklart finns inga monopoler eftersom inga poler alls existerar. Jag hävdar att alla magnetiska fenomen enkelt låter sig förklaras med Coulombs lag och relativistisk längdkontraktion. Tag t.ex två parallella motriktade elektriska stömmar. Ömsesidigt ser elektronerna en elektronvärld i grannens tråd som har längdkontraherats relativistiskt varför coulombrepulsion uppträder. I alla lägen kan magnetism förklaras genom två enkla principer: 1. Lika strömmar attraherar varandra, motriktade strömmar repellerar varandra där kraften är en relativistisk coulombkraft. 2. All s.k. magnetism uppkommer ur elektriska strömmar. Insisterar man med att införa begreppet magnetfält och magnetiska poler kan man ju försöka klargöra var polerna finns i det cirkulära magnetfältet kring en elektrisk ledare. Är jag helt fel ute här eller?
/Bernt S, Dahlstiernska gymnasiet, Mellerud

Svar:
Den generellt accepterade statusen vad gäller existensen av magnetiska monopoler är att inga teoretiska hinder finns för deras existens. Du kan kalla det fullständigt nonsens om du vill, men statusen är ett faktum. Vi som svarar i frågelådan kan inte allt, och det finns säkert felaktiga svar, men vi gör vårt bästa och försöker alltid svara så korrekt som möjligt.

Maxwells ekvationer (se bilden nedan och länk 1) sammanfattar de klassiska egenskaperna av elektromagnetism på ett utomordentligt kompakt sätt. Matematiken är inte helt lätt, men alla måste ändå slås av skönheten i dessa fyra ekvationer (i ordning uppifrån och ner):

Gauss lag för elektricitet
Faradays induktionslag
Gauss lag för magnetism (frånvaron av magnetiska monopoler)
Amperes lag

Den sista ekvationen är även grunden för elektromagnetisk strålning.

Vad gäller magnetiska monopoler (Magnetic monopole ) så är det mycket möjligt att de inte existerar. Ett skäl är att de om de existerade borde vara ganska lätta att detektera, men man har inte sett några indikationer i något experiment (Magnetic Monopole Searches ). Om magnetiska monopoler existerar måste naturligtvis den tredje ekvationen nedan modifieras.



/Peter E

Nyckelord: magnetism [52]; magnetisk monopol [4]; Maxwells ekvationer [3]; #ljus [63];

1 http://hyperphysics.phy-astr.gsu.edu/hbase/electric/maxeq.html
2 http://fy.chalmers.se/~tfymp/Homepage/Teaching/FFM232/lecture8.pdf

*

Ljud-Ljus-Vågor [13590]

Fråga:
Tja, undrar bara en sak vet inte om det finns något svar men har funderat på det ett tag. Hur ser fotonens och olika ljus vågrörelser ut är de bara åt 2 riktningar t.ex. upp och ner som vågor på vattnet eller sidleds också eller kanske möjligen skiftande?
/Karl j, Hjärteskolan, Trosa

Svar:
Ljus är en elektomagnetisk svängningsrörelse. Det finns ett elektriskt fält och vinkelrätt däremot ett magnetiskt fält som svänger med mycket hög frekvens. Båda svängningarna är vinkelräta mot fotonens rörelseriktning, se nedan.

Polarisation: Riktningen av det elektriska fältet definieras som polarisationsriktningen.

Polarisationsriktningen kan variera, men varje polarisationsriktning kan sammansättas av två komponenter: t.ex. en horisontell och en vertikal.

Man kan åstadkomma polariserad elektromagnetisk strålning på många sätt, t.ex. absorption (galler), reflektion (Brewster-vinkeln), sändarantennens utformning (radiovågor), comtonspridning (gammastrålning). Se Polarizer .

För en vattenvåg är bara en svängningsriktning möjlig - den riktning som är vinkelrät mot ytan - så för en sådan saknar det mening att tala om polarisation.



/Peter E

Nyckelord: polarisation [7]; elektromagnetisk strålning [21]; #ljus [63];

*

Ljud-Ljus-Vågor [13368]

Fråga:
Hur alstras det polariserade ljuset som kommer från himlen?
/Veckans fråga

Ursprunglig fråga:
Hur alstras det polariserade ljuset som kommer från himlen? Jag har studerat ljuset med hjälp av polarisationsfilter. Det är inte hela himlen som är polariserad utan polariseringen går som ett band över himlen, vinkelrätt mot solstrålarna, som ett bildäck. Det är tydligast på morgon och kväll, ett välkänt fenomen.

Min tolkning är följande. Det är det blå ljuset från himlen som är polariserat. Jag misstänker att det är syre, ozon eller kväve, alltså en molekyl med dubbel eller trippelbindningar, som svarar för polariseringen och skickar ut infallande ljus vinkelrätt mot infallsvinkeln. Om det infallande ljuset är blått, ultraviolett eller ännu kortvågigare har jag ingen idé om. Det blå ljuset på himlen kommer från ozon, men det är fluorescens. Det har nog inget med polariseringen av ljuset att göra.

Jag är pensionerad biologilektor från Polhemskolan här i Lund. Jag följer fågelsträcket i Falsterbo och läser om frågor om fåglarnas orientering, hobbyforskning. Fåglar uppfattar polariserat ljus. Läste på 70-talet 20 poäng fysik med resultatet mvg, och lite fysik sitter fortfarande kvar. Jag är tacksam för tips om litteratur om polarisering av himlen. Finns det någon som kan hjälpa mig? Mycket tacksam för någon form av svar.
/Ingvar l, f.d. Polhemskolan, Lund

Svar:
Ingvar! Vi har svarat flera gånger på frågan varför himlen är blå (se nedanstående avancerade sökning), men inte i detalj gått in på processen och varför det spridda ljuset är polariserat.

Anledningen till att himlen är blå är att solljuset (bestående av alla färger, dvs i princip vitt) sprids av luftens molekyler i en process som kallas Rayleigh-spridning (NE: spridning av ljus mot partiklar som är mycket mindre än ljusets våglängd, t.ex. luftens molekyler), se nedanstående figur. (Observera att solen är vit, inte gul!) Figuren ger också uttrycket för sannolikheten för spridning som funktion av spridningsvinkeln q och ljusets våglängd l.

Förutom några konstanter består spridningssannolikheten av två termer:

1/l4
Ljus av kort våglängd sprids allså mycket mer är ljus av lång våglängd. Om vi jämför blått ljus (4500 Å) med rött ljus (6500 Å) får vi förhållandet (6500/4500)4=4.4. Blått ljus sprids alltså betydligt mer än rött. Det är anledningen till att himlen ser blå ut.

(1 + cos2q)
Denna term säger att intensiteten av det spridda ljuset är minst 90o från solen. I själva verket är intensiteten i 90o precis hälften av intensiteten i 0o. Detta ger en indikation av vad som orsakar denna term: polarisationen. Ljuset från solen är opolariserat och kan ses som två polarisationsriktningar med samma intensitet vinkelräta mot varandra. I 90o kan endast den ena riktningen spridas. Intensiteten där blir alltså hälften. För att bekräfta teorin kan man kontrollera ljusets polarisation i olika riktningar (olika spridningsvinklar). Man finner då att polarisationsgraden är maximal 90o från solen.

Fåglar, som kan uppfatta polarisation utan hjälpmedel (t.ex. polarisationsglasögon) kan alltså bestämma riktningen mot solen även när denna skyms av moln.

Kan man förstå varför bara den ena polarisationsriktningen kan spridas i 90o? Ja, det är ganska lätt att ge en enkel bild av processen om man vet hur en dipol (enkel, rak sändarantenn) sänder ut strålning. En dipol strålar maximalt i 90o och inte alls i 0o.

Vi delar det infallande solljuset i två polarisationskomponenter - polarisationsriktningen för elektromagnetisk stålning är E-vektorns riktning: vinkelrätt mot synlinjen och parallellt med synlinjen. Ljus som faller in med polarisationsriktningen i synlinjen kommer att få elektroner att svänga i synlinjen. Dessa dipoler kan alltså inte stråla i synlinjen. Ljus som däremot faller in med polarisationsriktningen vinkerätt synlinjen kommer att få elektroner att svänga vinkelrätt mot synlinjen. Dessa dipoler kan alltså att stråla maximalt synlinjen. Alltså ser vi bara den senare hälften, och det spridda ljuset är polariserat.

Rayleigh-spridning är helt oberoende av vilka molekyler vi har eftersom alla molekyler är ungefär lika stora. Processen är alltså inte fluorescens (NE: en form av luminiscens [utsändande av ljus] från ett system) utan i princip elastisk spridning mot bollar utan inre struktur.

Se även snackset Varför är himlen blå? och nedanstående länk.



/Peter E

Nyckelord: blå himmel [12]; polarisation [7]; #ljus [63];

1 http://hyperphysics.phy-astr.gsu.edu/hbase/atmos/blusky.html

Avancerad sökning på '"himlen blå"' i denna databas

*

Ljud-Ljus-Vågor [13218]

Fråga:
Hej. Finns det någon bra demonstration/försök för en gymnasieklass att visa att ljushastigheten är konstant, oberoende av ljuskällans rörelse.
/Külvert J

Svar:
Külvert! Det är en ganska besvärlig fråga eftersom ljushastigheten i vakuum c är fundamentalt involverad i relativitetsteorin. Det korta svaret är nej det finns inget enkelt försök. Eftersom problemet är så grundläggande kan det vara på sin plats att diskutera det lite.

Att c är konstant oberoende av källans eller mottagarens rörelse är ett postulat (fundamentalt antagande) i Einsteins speciella relativitetsteori. Einstein grundade antagandet på resultatet av ett experiment där Michelson-Morley försökte påvisa etern. Etern vad det medium man trodde transporterade elektromagnetisk strålning. Resultatet av M-M försök var att de inte såg någon skillnad på olika riktningar trots att jorden rör sig i sin bana med en hastighet av 30 km/s. M-M tolkade resultatet så att jordens rörelse genom etern "tryckte ihop" jorden, så att det inte blev någon skillnad. Se vidare Michelson-Morley Experiment och Flash-animering på Michelson-Morley Experiment (Flash) . Observera att den senare visar vad som skulle hända om etern fanns.

Einstein omtolkade resultatet av M-Ms experiment så att han i stället antog att c är konstant oberoende av källans eller mottagarens rörelse. Från detta antagande härledde han en teori - den speciella relativitetsteorin - som i nästan 100 år visat sig stämma exakt med alla mätningar. Ljushastighetens konstans kan alltså anses väl etablerad.

Så etablerad att den internationella unionen för mått och vikt 1983 antog en ny definition av längdenheten meter, enligt vilken en meter är den sträcka som ljuset tillryggalägger i tomrum på 1/299 792 458 sekund. Därmed är ljushastigheten i vakuum definierad till exakt 299 792 458 m/s. Se vidare Is The Speed of Light Constant? , Speed_of_light och The Speed of Light .

M-Ms experiment med interferometrar i två vinkelräta riktningar visar (om man tolkar det så) att ljushastigheten är oberoende av mottagarens rörelse. Om man noggrannt observerar hur dubbelstjärnor rör sig i sina banor kan man även visa att ljushastigheten är oberoende av källans rörelse.

Så som du ser var det ingen lätt fråga du ställde . Tack Elisabeth och P-O för synpunkter !

Fotnot:

Experiment att bestämma ljushastigheten som är lätt att förstå men kräver lite utrustning finns i länk 1 nedan.
/Peter E

Se även fråga 3545 och fråga 12753

Nyckelord: elektromagnetisk strålning [21]; ljushastigheten [24]; relativitetsteorin, speciella [45]; #ljus [63];

1 http://www.csc.kth.se/~tomaso/ufu/ljusexperiment.html

*

Ljud-Ljus-Vågor [12865]

Fråga:
Kan man bli brun om man sitter innanför en vanlig glasruta? Vilka strålar, UV-A UV-B UV-C, går igenom glas och vad gör de olika strålarna?
/Roger S, Berzeliusskolan, Linköping

Svar:
Roger! Det måste vara vår i luften nu när vi får in frågor om solbränna...

Det ultravioletta ljuset kan delas in i tre våglängdsområden: UV-C mellan 100 och 280 nm, UV-B mellan 280 och 315 nm och UV-A med våglängder mellan 315 och 400 nm. Som jämförelse kan nämnas att det för människan synliga ljuset ligger mellan 380-780 nm. Enligt Länk 1 absorberas all UV-C från solen i atmosfären, och UV-B absorberas till stor del av ozonlagret. UV-A absorberas dock nästan inte alls av atmosfären.

Enligt länk 2 är det UV-B som som effektivast bildar pigment i huden, om pigmentet därefter utsätts för UV-A så uppkommer solbränna.

Vanligt fönsterglas absorberar i stort sett all strålning med våglängder under 300 nm, vilket alltså innebär att det stoppar UV-B och UV-C, men släpper igenom UV-A. Från detta drar jag slutsatsen att om huden i förväg utsatts för UV-B, och därmed bildat pigment, bör man kunna bli brun "i efterhand" även om man sitter bakom fönsterglas!

I sammanhanget är det intressant att veta att ljuset från lysrör i solarier domineras av UV-A, vilket gör att man bygger upp ett mycket sämre solskydd mot UV-B som är farligast för huden i ett solarium. Om huden inte fått bygga upp ett bra skydd mot UV-strålning, men trots detta utsätts för starkt solljus, kan den inte absobera UV-ljuset och resultatet blir ett inflammationstillstånd med rodnad, ömhet och i vissa fall även blåsbildning - "soleksem". Så småningom fjällar huden av, och har man otur och är överkänslig samt utsätter sin hud för upprepade solskador i huden så kan hudcancer uppkomma. (Från länk 2.)



/Margareta H/lpe

Nyckelord: UV-ljus [13]; lysrör [10]; strålning, faror med [26]; #ljus [63];

1 http://www.stralsakerhetsmyndigheten.se/start/Sol-och-solarier/njut-av-solen/UV-stralning/
2 http://www.reseledaren.nu/solskyddsmedel.asp

*

Värme [12852]

Fråga:
Vi är två NO-kollegor som vill förklara detta rätt. Har hittat aspekter av svaret, men inte något med direkt fokus på denna fråga: Varför är stearinlågan blå allra längst inne vid veken?

Har det bara att göra med att elektronerna exiterar pga av värmen när stearinvätskan i veken övergår i gasform?

Jag och en kollega har olika uppfattningar och det handlar som du säkert som god naturvetare kan inte om att ha rätt eller fel utan en ren nyfikenhet. Naturligtvis handlar det också om att visa för eleverna att två helt olika teorier kan förklara exakt samma skede.

Min fundering handlar också om att det också finns med fullständig förbränning längst inne vid veken. En skorstenseffekt helt enkelt. Att det blir några kanaler som leder upp syre längs med veken (undertryck?).

Vid den fullständiga förbränningen bildas avgaser och då så blir det en mer ofullständig förbränning. Därav lågans mer gula sken längst ut.

Om detta inte ligger inom ditt arbetsområde så får du gärna förmedla vidare frågorna. Jag ska också undersöka andra internetresurser.
/Petter G, Furulunds skola Lunden, Partille

Svar:
Petter! Det kan tyckas märkligt att lågan är blå nere vid veken. Blått uppfattas ju som varmare än t.ex. gult. Detta gäller emellertid endast om det är svartkroppsstrålning. I en ljuslåga kommer det blå från band från molekylerna C2 och CH.

Det gula i en stearinljuslåga kommer från strålning från kolpartiklar (sot) som bildas av en ofullständig förbränning. Det gula området är ett mycket tunnt skal (någon millimeter) där bränslet (det förångade paraffinet) möter syret. Strålningen är en mycket komplicerad blandning av linjer och band, och är inte enbart svartkroppsstrålning (gult motsvarar - tänk på solen - av c:a 5500oC och så varm blir inte en ljuslåga).

En bunsenbrännare är bra för att demonstrera ovanstående effekt eftersom man med en sådan kan variera lufttillförseln. Med lufttillförseln strypt får man en fladdrande, gul och sotande låga (ofullständig förbränning), medan man med lufttillförseln öppen får en helt blå låga (fullständig förbränning). Se Bunsen_burner#Operation .

Bilden nedan, från Causes of Color , visar en vanlig ljuslåga till vänster. Denna karakteriseras av en luftström nerifrån och upp (varm luft stiger i ett tyngdkraftfält). Lågan till höger är en låga i tyngdlöst tillstånd i en rymdstation. Vi ser att lågan är rund (ingen strömning), och bara blå. Uppenbarligen bildas inte sot. Lågan slocknar efter en kort stund eftersom inget syre tillförs genom strömning.

Se även en utmärkt sajt om färger: Causes of Color och nedanstående länk.

Tack Johan Zetterberg, Förbränningsfysik, för att du förklarat ljuslågans fysik för mig!



/Peter E

Nyckelord: stearinljuslåga [16]; #ljus [63];

1 http://www.astronomycafe.net/qadir/q490.html

*

Ljud-Ljus-Vågor [12739]

Fråga:
jag vet inget om fiberoptik men vill gärna veta lite om detta! kan ni berätta något?
/joel o, nyh, nyh

Svar:
Joel! Motiveringen bakom utvecklingen av fiberoptik var att man ville kunna sända ljuspulser över mycket långa avstånd utan att signalen går förlorad på vägen. Att sända ljussignaler genom luft över långa sträckor är ingen säker metod - t.ex. om det regnar eller någon ställer sig i vägen bryts förbindelsen, och även om det inte sker, så sprids ljuset mot luftmolekyler och signalen blir svagare och svagare.

Sedan länge har man undersökt hur ljus bryts och reflekteras i linser och speglar, så man vet att under vissa förhållanden kan man få en ljusstråle som skickas in i ett speciellt utformat prisma att "stanna" inuti prismat - varje gång ljusstrålen träffar på en av prismats ytor totalreflekteras det och tränger inte ut. Man kan säga att fiberoptiken föddes när man fick möjlighet att tillverka mycket långa och smala "prismor" - vad vi kallar optiska fibrer.

Hur åstadkommer man detta praktiskt? Som du kanske redan vet, är det ett villkor för att ljus överhuvudtaget ska kunna brytas eller reflekteras när det träffar en gränsyta mellan två material - t.ex. luft och glas, att dessa har olika brytningsförmåga, eller som vi fysiker säger, brytningsindex. Det som nu avgör om en inkommande ljusstråle reflekteras eller "bara" bryts i gränsytan är dels infallsvinkeln och dels förhållandet mellan materialens brytningsindex.

Om ljuset kommer från ett material och möter ett som har lägre brytningsindex kan totalreflektion uppstå om vinkeln är tillräckligt stor - det betyder att allt ljus reflekteras i gränsytan. (Om vinkeln är mindre kommer endast en del av ljuset att reflekteras, resten tränger in genom ytan.) Detta faktum använder man sig av när man konstruerar en optisk fiber. Fiberns kärna är en smal (typiskt är diametern 40-70 mikrometer) cylinder, oftast gjord av en speciell sorts glas, med ett visst brytningsindex. Utanpå kärnan lägger man ett skikt av ett annat material med ett lägre brytningsindex, se nedanstående figur från Wikimedia Commons. Nu är fibern i princip klar att användas, men oftast lägger man på ytterligare ett skikt av plast för att skydda den ömtåliga fibern mot fukt, repor etc.

Om man nu skickar in ljussignaler i fibern med en tillräckligt stor vinkel kommer ljuset att reflekteras varje gång det träffar gränsytan mellan kärnan och det utanpåliggande skiktet. I princip skulle man på detta sätt kunna skicka en ljussignal hur långt som helst. I praktiken uppstår dock alltid förluster - en del av ljuset går förlorat genom absorption och spridning i det material fiberns kärna är gjort av, och dessutom är reflektionen i en verklig fiber inte 100%-ig (men bra nära!).

Artikeln fiberoptik ger mer information om olika typer av optiska fibrer och hur man kan använda fiberoptik i praktiska tillämpningar. Fiberoptik är bra men Optical_fiber är mer detaljerad (engelska).

CK Kao (Charles_K._Kao ) delade nobelpriset i fysik 2009 för sin upptäckt och utveckling av fiberoptik, se länk 1.



/Margareta H/lpe

Nyckelord: fiberoptik [4]; #ljus [63];

1 http://nobelprize.org/nobel_prizes/physics/laureates/2009/sci.html

*

Ljud-Ljus-Vågor [12631]

Fråga:
Varför ser vi saker i speglar och linser?
/Veckans fråga

Ursprunglig fråga:
Varför ser vi saker i speglar och linser?
/Elisabet Å, skarpäng, täby

Svar:
Jag tolkar frågan så, att du vill veta hur det kommer sig att ljus som träffar en spegel reflekteras eller ljus som går igenom en lins kan t.ex. fokuseras eller böjas av?

Ljus kan beskrivas som en elektromagnetisk vågrörelse. När en sådan ljusvåg kommer in mot en yta av något material kan olika saker hända: den kan reflekteras, absorberas eller böjas av... Vad som sker beror bl.a. på ytans beskaffenhet (slät eller skrovlig), ljusets färg och infallsvinkel, och förhållandet mellan brytningsindex hos linsen och omgivningen (oftast luft). Ett materials brytningsindex talar om hur snabbt ljuset kan ta sig fram i materialet, och har att göra med materialets inre struktur och sammansättning. Luft har brytningsindex 1.0, medan vatten har ca 1.3 och glas 1.5-1.6.

Linser består oftast av glas eller plast som formats på ett speciellt sätt så att de får "runda" ytor. T.ex. kan de vara konvexa eller konkava - formade som utsidan eller insidan på en kupad hand. Formen har stor betydelse för vinkeln mellan först det inkommande ljuset och ytan, och sedan för den vinkel gentemot ytan på linsens baksida där ljuset kommer ut.

Se vidare länken nedan.

Vanliga speglar fungerar på ett litet annorlunda sätt. De består oftast av en klar glasskiva med ett tunt metallskikt på baksidan. Glaset är till för att skydda metallytan - det är den som är den egentliga spegeln. Metallen innehåller "fria" elektroner (som inte är bundna till någon speciell metallatom), och när ljusvågorna växelverkar med elektronerna i metallytan blir effekten att nästan inget ljus kan tränga in i metallen - det reflekteras istället. Om spegelytan är plan ser man en tydlig spegelbild, men om ytan är skrovlig eller ojämn kastas det reflekterade ljuset ut i olika riktningar vilket gör spegelbilden otydlig.

För att beräkna vad som händer med speglar och linser använder man sig av stråloptik. Man följer då den väg ett litet antal strålar tar, och man kan på så sätt få fram hur bilden ser ut. Se vidare PowerPoint presentationen Stråloptik från Malmö högskola. Bilden nedan visar vad som händer med en positiv lins.



/Margareta H/Peter E

Nyckelord: ljusbrytning [26]; spegel [10]; lins [11]; #ljus [63];

*

Elektricitet-Magnetism [12571]

Fråga:
Hur fungerar lysdioder med vitt ljus?
/Veckans fråga

Ursprunglig fråga:
Det blir allt vanligare med vita lysdioder i olika sammanhang. Hur är de konstruerade för att kunna ge ett så intensivt sken? Förklarar konstruktionen det höga priset?
/Mats A, Stockholm

Svar:
Funktionaliteten hos en lysdiod beskrivs bra i Dioder . Wikipedias artikel Led innehåller mycket information och bra länkar.

För att få vitt ljus kan man ha flera lysdioder av olika färg (röd, grön, blå). Dessa kan då blandas till en godtycklig färg, inklusive vitt.

För att ersätta glödlampor använder man ofta fluorescens för att göra om blått ljus (som har det högsta energiinnehållet) till andra färger. Detta är samma metod som används i lysrör och lågenergilampor. Bilden nedan (från Wikipedia-artikeln ovan) visar hur spektrum ser ut. Den smala toppen vid 450 nm är det blå ljuset från dioden och den breda fördelningen vid längre våglängder från fluorescensen gör att ljuset uppfattas som någotsånär vitt.

Det finns två fördelar med att använda lysdioder för belysning. För det första håller de mycket längre (c:a 100000 timmar jämfört med c:a 1000 timmar för en glödlampa och c:a 10000 timmar för ett lysrör). För det andra går nästan 100% av den elektriska energin till produktion av ljus. För en glödlampa är denna effektivitet mindre än 5% - mer än 95% blir värme. För fluoriserande vita lysdioder har man lite förluster som uppkommer när man skiftar det blå ljuset till ljus med lägre energi (längre våglängd) - effekten kallas Stokes-förskjutning, se Stokes_shift .

Vad gäller priset så finns nog ett antal förklaringar. Dels är lysdioder som ger blått ljus (som behövs för att göra vitt ljus) svåra att tillverka. Lysdioder tillverkas under extremt rena förhållanden, för att materialen inte skall kunna ta skada av störande partiklar som försämrar deras funktion. Sedan är produkterna relativt nya - med mer konkurrens kommer de säkert att bli billigare. Dessutom kommer man säkert snart på hur man skall tillverka dem billigare.

PS 7/10/2014:
Nobelpris i fysik 2014 för blå lysdioder, se länk 1.



/Peter E

Se även fråga 1677

Nyckelord: Illustrerad Vetenskap [17]; lysdiod [14]; glödlampa [23]; fluorescens [6]; #ljus [63];

1 http://www.nobelprize.org/nobel_prizes/physics/laureates/2014/press.html

*

Ljud-Ljus-Vågor [12409]

Fråga:
Varför är solen gul när den är mycket varmare än en gaslåga som är blå?
/Veckans fråga

Ursprunglig fråga:
Varför är solen gul när den är mycket varmare än en gaslåga som är blå? Blått är ju varmare än gult? Har det med att göra hur långt elektronerna hoppar?
/Catharina R, Sollentuna musikklasser, Sollentuna

Svar:
Bra fråga Catharina!

Solens "yta" (fotosfären) sänder ut temperaturstrålning, dvs elektromagnetisk strålning som utsänds från varje kropp med temperatur över absoluta nollpunkten. Maximum för denna fördelning ligger i gult för solytans temperatur, c:a 6000 grader. Temperaturstrålningen för en s.k. absolut svart kropp (en kropp som absorberar all strålning som kommer in) beror bara på temperaturen, inte sammansättningen. Spektrum för temperaturstrålningen visas nedan för några temperaturer; figuren kommer från Radiation Laws .

Genom att mäta upp vid vilken våglängd maximum ligger, kan man bestämma temperaturen hos en kropp. Det är så man bestämt solytans temperatur till c:a 6000oC.

En gaslåga sänder ut ett linjespektrum, dvs ett spektrum som består av spektrallinjer. Vilka linjer som utsänds beror på atomernas egenskaper. Så färgen på gaslågan beror på atomernas energinivåer.

Försök: tänd en bunsenlåga och justera den så den är blå. Kasta lite koksalt i lågan. Vad händer? Effekten beror på natriumet i NaCl.

Se vidare temperaturstrålning och linjespektrum i Nationalencyklopedin . Temperaturstrålning (även kallad svartkroppsstrålning, ett begrepp som är förvirrande) behandlas även i Svartkropp .



/Peter E

Nyckelord: spektrum [11]; Plancks strålningslag [6]; temperaturstrålning [29]; #ljus [63];

*

Materiens innersta-Atomer-Kärnor [12397]

Fråga:
Hur kan man härleda Plancks strålningslag?
/Veckans fråga

Ursprunglig fråga:
Hur kan man härleda plancks strålningslag med hjälp av fysiken från fysik A kursen eller åtminstone så att vi som bara läst fysik B i ett halvår förstår det.
/Niclas B, Stockholm

Svar:
Det kan man inte. Planck själv förstod den inte . Jag bara skojar det gjorde han naturligtvis, men han tyckte inte om den.

För att kunna förklara intensiteten vid korta våglängder (ultraviolettkatastrofen1 i den klassiska teorin, se Ultraviolet_catastrophe ), var han tvungen att postulera att strålningen bara kunde utsändas i speciella kvanta med energin

E = hv

där h är Plancks konstant och v är strålningens frekvens.

På så sätt fick han fördelningen att gå mot noll även för korta våglängder, se nedanstående figur. Det finns en ganska bra härledning i Krane, Modern Physics. ISBN: 0-471-82872-6 (McMurry, Quantum Mechanics ISBN 0-201-54439-3).

Plancks strålningslag (Planck's_law ) är fördelningen i våglängd hos en svartkroppsstrålare:

där B är utstrålad effekt, T är svartkroppens absoluta temperatur, kB är Boltzmanns konstant, h är Plancks konstant och c är ljushastigheten.

När man fått fram uttrycket för Plancks strålningslag är det relativt lätt att med hjälp av ett matematikprogram (Maple eller Mathematica) härleda Stefan–Boltzmanns lag (Stefan-Boltzmann_law )

P = e s*T4

genom att integrera Planck-kurvan. T är absoluta temperaturen, e är emissiviteten och

s = 5.67 10-8 J s-1 m-2 K-4

är Stefan-Bolzmanns konstant.

Wiens förskjutningslag (Wien's_displacement_law )

lmax*T = 2.898×10−3 m·K

härleds genom att derivera med avseende på våglängden och sätta derivatan lika med noll för att få maximum.

Se även fråga 14668 och Black Body Radiation .

_______________________________________________________________
1 Man ser i nedanstående figur att problemet med den klassiska teorin är att strålningen för höga frekvenser (korta våglängder) överskattas våldsamt. Anledningen är att det klassiska teorin bara tar hänsyn till att det finns fler tillstånd med korta våglängder än långa (kort våglängd betyder att fler stående vågor får plats i kaviteten).

Plancks antagande att energin hos en foton (ett begrepp som senare infördes av Einstein) beror av frekvensen (E=hv) betyder att det "kostar" energi att göra höga frekvenser. Vi får då en dämpande faktor given av boltzmannfördelningen (Boltzmannfördelning )

e-E/kT.

Med denna korrektion får man ovanstående planckfördelning vilken stämmer mycket bra med experimentella observationer.



/Peter E

Nyckelord: Plancks strålningslag [6]; #ljus [63];

1 http://physics.info/planck/
2 http://www.fysik.su.se/~milstead/fyu02/lec01_sv.pdf

*

Blandat [12321]

Fråga:
Varför är katter gråa i mörkret?
/Veckans fråga

Ursprunglig fråga:
Varför är katter gråa i mörkret? Varför ska man fokusera sin blick vid sidan av det föremålet man vill se på i mörker?
/stina a, fristadshögstadie, fristad

Svar:
Det beror på hur ögat är konstruerat. Synceller består av två typer: tappar som finns i tre varianter och kan se färger och stavar som inte ser färg men som är känsligare.

På natten fungerar bara stavarna eftersom tapparna kräver mer ljus. Du är därför färgblind på natten.

Svaret på din andra fråga är att det centralt på näthinnan finns mycket tappar och färre stavar. Detta för att ge bra färgseende. I utkanten av näthinnan finns många stavar, dvs mörkerseendet är bättre där.

Observera att stavarna är inaktiverade när belysningen är god. Det tar flera minuter i mörkret innan du ser bra. Detta kallas ögats mörkeradaption. Det finns andra intressanta egenskaper vad gäller synfunktionen hos människan, se förklaringen till Machs band i Mach bands .

Man kan fråga sig varför ögat av evolutionen utvecklats på detta sätt med stavar som är färgblinda. Det beror ganska säkert på att stavarna kan göras känsligare (är mer effektiva detektorer) eftersom de kan detektera allt infallande ljus medan tapparna bara detekterar ljus av en viss våglängd.
/Peter E

Se även fråga 3139

Nyckelord: färg/färgseende [39]; *vardagsfysik [64]; #ljus [63];

*

Ljud-Ljus-Vågor [12202]

Fråga:
Varför ser det ut som ränder mellan fingrarna när man håller upp två fingrar mot en ljuskälla.
/Veckans fråga

Ursprunglig fråga:
Varför ser det ut som ränder mellan fingrarna när man håller upp två fingrar mot en ljuskälla. Jag är ute efter det vetenskapligt korrekta svaret.
/Aneth S, Lärarutbildningen Högskolan i Borås, Borås

Svar:
Denna fråga orsakade mycket bryderi! Ränder eller fransar får en fysiker omedelbart att tänka på interferens. Men det är antagligen inte korrekt (din begäran av det vetenskapligt korrekta svaret skrämmer oss .

Anledningen till att vi inte tror att det är interferens är att effekten är oberoende av ljuskällan. Man skulle vänta sig att en laserljuskälla skulle visa effekten tydligare än t.ex. solen eller en glödlampa. Dessutom, om man får interferensfransar med en glödlampa, så skulle man vänta sig att de olika färgerna skulle vara förskjutna i förhållande till varandra. Det mönster man ser är emellertid bara gråtoner.

Man kan se effekten ganska bra på följande enkla vis: Skym en ljuskälla med ett finger. Nära fingret (som inte skall vara belyst från det håll ögat är) kan du se grå ränder. Effekten syns egentligen mer generellt än så: vid alla skarpa övergångar från ett ljust fält till ett mörkt fält.

Det du ser är ganska säkert Machs band (eng. Mach bands). Effekten upptäcktes av Ernst Mach (samma gubbe vars namn givit namn år en hastighetsenhet mach 1 = ljudhastigheten). Effekten har att göra med hur sensorerna i ögat är konstruerade. Det är inte så enkelt att utsignalen till hjärnan är direkt relaterad till insignalen (ljusintensiteten) i en punkt. Varje sensor är omgiven av en inhibitor (undertryckande sensor).

Ögat har ett fantastiskt dynamiskt område (fungerar bra både i starkt och svagt ljus). Till en del beror detta på att bländaren (pupillen) ändras, men också på de ovan beskrivna inhibitorerna.

För en mer detaljerad förklaring se Mach bands eller sök på 'Mach bands' på webben. Tack bland annat Sven-Göran, Elisabeth och Göran för funderingar och diskussioner!

Tack Aneth och Karin för en rolig och utmanade fråga!
/Peter E

Nyckelord: #ljus [63];

*

Ljud-Ljus-Vågor [12222]

Fråga:
Varför visar skuggan av ett ljus både ljuset och en skugga av en "rök"?
/Veckans fråga

Ursprunglig fråga:
Varför visar skuggan av ett ljus både ljuset och en skugga av en "rök" trots att jag inte ser röken när jag tittar på ljuset?
/Mia G

Svar:
Mia! Mycket intressant observation! Jag har utfört följande experimentet med hjälp av min assistent (frugan):

Placera ett ljus på ett bord och placera en lampa så att skuggan av ljuset faller på väggen. Tänd ljuset. Man ser då förutom skuggan av ljuset även skuggan av något som verkar vara rök ovanför ljuset. Men om man tittar direkt på ljuset syns ingen rök! Man får en tydligare effekt om man fläktar lite så att ljuslågan rör sig. Vad är det man ser?

Förklaringen är att den varma luften som stiger från ljuset har lägre densitiet (är tunnare) än resten av luften. Ljus som passerar områden med varierande densitet bryts och reflekteras. Föreställ dig en ljusstråle som går från lampan, genom området ovanför ljuset och fram till väggen. En liten del av detta ljus sprids/reflekteras så det inte hamnar där det skulle. Det blir alltså lite mörkare, och denna "skugga" uppfattas som rök.

Detta är samma effekt du kan se på en uppvärmd väg eller i öknen. Luftspegling och hägringar beror på samma effekt: brytning/totalreflektion i områden där luftens densitet ändras.
/Peter E

Se även fråga 629

Nyckelord: stearinljuslåga [16]; #ljus [63];

*

Ljud-Ljus-Vågor [12225]

Fråga:
Hur tar polaroidglasögon bort reflexer?
/Veckans fråga

Ursprunglig fråga:
När ljus reflekterat svänger det bara i ett plan och därför kan vi slippa reflexer med polaroidglasögon. Jag förstår hur glasögonen fungerar, men inte varför reflexionen gör ljuset polariserat. Hoppas ni kan hjälpa mig med det.
/Eva B, Sandagymnasiet, Huskvarna

Svar:
Vid reflexion ändras både ljusets intensitet och polarisation. När en ljusstråle träffar gränsytan på ett genomskinligt medium kommer en del av ljuset att brytas och en del av ljuset att reflekteras. Om ljuset faller in mot gränsytan så att den brutna strålen och den reflekterade strålen är vinkelräta mot varandra, så kommer det reflekterade ljuset att vara planpolariserat. Infallsvinkeln som uppfyller sambandet, och alltså ger 100% polariserat reflekterat ljus, kallas för brewstervinkeln.

Om det i brewstervinkeln infallande ljuset är planpolariserat vinkelrätt mot infallsplanet kommer allt ljus att brytas ner i mediet med högre brytningsindex och inget ljus reflekteras.

Vi tar en mycket enkel modell av spridningen, se nedanstående figur: den inkommande vågens E-fält (som definieras som polarisationsriktningen) sätter elektroner i svängning i gränsytan. Komponenten som är parallell med normalen får elektronerna att svänga nästan i den riktning som den reflekterade strålen går (exakt för Brewstervikeln). En dipol strålar inte i denna riktning. Det betyder att den med infallsplanet parallella komponenten inte reflekteras, utan måste gå in i mediet. Den andra komponenten - som reflekteras utmärkt - tas om hand av polaroidglasögonen. Alltså ser vi inget reflekterat ljus.

Se vidare Brewster's_angle .



/Peter E

Nyckelord: polaroidglasögon [3]; *vardagsfysik [64]; polarisation [7]; #ljus [63];

*

Ljud-Ljus-Vågor [12208]

Fråga:
Jag undrar hur farlig ultraviolet strålning är. Vad beror fluorecens och fosforecens på?
/Veckans fråga

Ursprunglig fråga:
Hej. Jag undrar hur farlig ultraviolet strålning är. Jag letar mineral och även fluoricerande och jag har då en UV-lampa till hjälp. Men det är väl skadligt för synen och det orsakar väl hudcancer? Men man har ju sådana på till exempel discon så hur farligt är det egentligen?

Vad beror för övrigt Fluorecens och Fosforcens på? Det låter som Fluor och Fosfor men långt ifrån alla till exempel fluoricerande mineral innehåller just Fluor. Väldigt bra sida/Thor
/Thor P, Valla, Katrinaholm

Svar:
Hej Thor! Mycket intressanta frågor du ställer!

Ultraviolett strålning är elektromagnetisk strålning vars våglängd är kortare än det synliga ljusets, se nedanstående figur. Den som utsätts för strålning kan drabbas av så kallad snöblindhet (se Snöblindhet ) och långvarig överexponering är en stor riskfaktor för hudcancer (se Malignt_melanom ).

Farligheten hos UV-strålning beror helt på hur stark lampan är och hur du riktar den. Om du riktar den mot marken och inte mot ansiktet är det inte farligt om det är en standardlampa som säljs fritt i Sverige. UV-ljuset reflekteras ganska dåligt.

Fluorescens och fosforescens är nästan samma sak, men fosforescens har större fördröjning. Vad som sker är att UV-ljus exciterar atomer till ett exciterat tillstånd. Detta sönderfaller genom att sända ut ljus av längre våglängd (mindre energi).

Fluorescens kommer av fluorit (flusspat, CaF2) som uppvisar effekten. Fosforescens kommer av fosfor eftersom en viss sorts fosfor är fosforiserande.

För mer information, se artiklarna i Wikipedia Fluorescence och Phosphorescence .



/Peter E

Se även fråga 3834 och fråga 2439

Nyckelord: UV-ljus [13]; fluorescens [6]; #ljus [63];

*

Energi [723]

Fråga:
Är det miljövänligt att använda lågenergilampor?
/Veckans fråga

Ursprunglig fråga:
Minskar jag den totala energikostnaden (inklusive kostnader för glödlampor och elförbrukning) om jag skaffar mig lågenergilampor till en villa med direktverkande el?
/Karin O, Mörarp, Mörarp

Svar:
En mycket bra fråga, som är ett av många exempel på att man, om man vill värna om vår miljö, måste ta hänsyn till alla effekter. Tåg är ju till exempel ett mycket energieffektivt och miljövänligt transportmedel, men man måste också ta hänsyn till exempelvis tunnelbyggande (tänk på Hallandsåsen!) och hur man producerar den ström som tågen behöver.

Lågenergilampor är små kompakta lysrör, som i stället för en glödtråd har en gas i vilken det blir en urladdning. Lågenergilampor innehåller miljöfarliga ämnen (t.ex. kvicksilver), så när de går sönder måste de tas om hand på ett bra sätt.

Lågenergilampor är betydligt dyrare än vanliga lampor, men de varar längre, så kostnaden blir ungefär densamma. Det har ingen betydelse om värmen kommer från lampor eller element - all energi från lamporna utom den lilla del som går ut genom fönstren används till uppvärmning. Dessutom är det ju så att vi behöver mest uppvärmning på vintern när vi även behöver mest ljus. Värmen från utspridda lampor blir även jämnare fördelad i rummet än från enstaka element.

Skulle man då lika gärna kunna använda bara lampor för uppvärmningen? Nej, det skulle bli för dyrt. Visserligen blir energiförbrukningen (det vill säga elräkningen) densamma, men lamporna går sönder efter en tid, medan ett el-element kan användas mycket länge. Dessutom blir det väldigt ljust hela natten, vilket kan vara störande.

Se även Lågenergilampor sprider kvicksilver med soporna .

Ljuskällor

Ljuskällor är konstruktioner som genererar ljus. Det finns flera olika ljuskällor som bygger på varierande fysikaliska fenomen. Nedan är en sammanställning av fördelar och nackdelar med olika ljuskällor. Detaljer om hur de olika ljuskällorna fungerar finns under länk 1, i Wikipedia-artiklarna Glödlampa och Lågenergilampa samt under nyckelordslänkarna nedan.

Eftersom olika ljuskällor skapar ljus på olika sätt (se nedan) så är fördelningen av ljuset med olika våglängd (spektrum) mycket olika, se figuren nedan.

Glödlampa
Mycket gammal och etablerad ljuskälla som är på väg att fasas ut genom EU-bestämmelser (Incandescent_light_bulb ). Glödlampan har en glödtråd (vanligen av wolfram) som ljuskälla. Glödtråden omges av en skyddande gas (vanligen en ädelgas) innesluten i en glaskolv. Glaskolven är fastsatt i en metallsockel, och elektriskt ledande trådar går från glödtrådens ändar ut genom sockeln. Glödtråden fungerar som ett elektriskt motstånd, och då ström leds genom den blir den het och fungerar därmed som en svartkroppsstrålare (temperaturstrålning ) som avger ljus och värme.
++ billig
+ behagligt, varmt ljus
- dålig ljuseffektivitet, c:a 2% (ger mycket värme, Incandescent_light_bulb#Efficiency_comparisons )

Lysrör
Etablerad ljuskälla i vissa miljöer, speciellt offentliga (Fluorescent_lamp ). Lysrör är en elektrisk ljuskälla som joniserar argon och kvicksilverånga för att bli elektriskt ledande varvid UV-ljus skapas. UV-ljuset exciterar elektroner i det fluorescerande pulver som är anbragt på glasrörets insida. När den exciterade elektronen återfaller i sin lägre bana avges synligt ljus. Glaskvaliteten i röret förhindrar UV-läckage.
+ relativt billigt
+ bra ljuseffektivitet (c.a 10 gånger bättre än en glödlampa)
+ lång livslängd
+ ljuset kan anpassas för olika behov med olika fluoriscenter
- startar upp långsamt (sekunder)
- kallt ljus
- svåra att reglera ljusstyrkan ("dimma")

Lågenergilampa
Är i princip ett litet lysrör och har därför liknande fördelar och nackdelar (Compact_fluorescent_lamp , Lågenergilampa )
+ relativt billig
+ bra ljuseffektivitet
+ lång livslängd (varierar dock)
- startar upp långsamt (sekunder)
- kallt ljus
- innehåller kvicksilver
- åldras (effektiviteten försämras)
- temperaturkänslig
- kan ej "dimmas"

Halogenlampa
Används mest i tillämpningar där man vill ha en liten ljuskälla, t.ex. bilstrålkastare, spotlight (Halogen_lamp ). En halogenlampa är en slags glödlampa som innehåller halogengas, vanligen av jod eller brom. I halogenlampan är temperaturen hos glödtråden högre än i en vanlig glödlampa. Den höga temperaturen gör att glödtråden förgasas till viss del men de gasformiga atomerna sublimerar och återgår till glödtråden i ett kretslopp så att den håller 2-4 gånger längre och ger mer ljus än en glödlampa. Halogenlampans ytterhölje är gjort av värmetåligt kvartsglas och har vanligen ett UV-skydd för att filtrera bort UV-strålning.
+ lite högre ljuseffektivitet än en vanlig glödlampa pga högre temperatur
+ innehåller inga tungmetaller
- relativt dyr
- kräver normalt lågspänning så man behöver en transformator

Lysdiod
Har snabbt etablerat sig som en mycket viktig ljuskälla när problemen högt pris och onjutbart ljus har lösts (Light-emitting_diode ). Lysdiod (Light Emitting Diode, LED) är en diod som utstrålar inkoherent monokromatiskt ljus vid en elektriskt framåtriktad spänning. Till skillnad från glödlampor, som kan använda likström eller växelström så kräver lysdioder likström med rätt polaritet (några få volt). När spänningen genom PN-övergången är i rätt riktning, flyter en betydande ström genom dioden. Strömmen säges då vara framåtriktad. Spänningen över lysdioden är i detta tillfälle stabil för en given lysdiod och proportionell mot energin av de utstrålade fotonerna. Speciella konstruktioner (fråga 12571 ) kan ge vitt ljus.
+ bra ljuseffektivitet
+ ej stötkänslig
+ mycket lång livslängd även om den tappar i effektivitet
+ innehåller inga tungmetaller
+ kan göras mycket små
+ ljuset kan fokuseras till en stråle utan reflektor
-- mycket dyr än så länge
- kallt ljus
- temperaturkänslig (speciellt för värme)

Översikt över de vanligaste lamptyperna (från Sydsvenska Dagbladet 5/12/09):

Glödlampa   Lågenergilampa   Halogenlampa   LED-lampa
   60W           15W            35W         7-15W
710 lumen      710 lumen       710 lumen   710 lumen
12 lumen/W     50 lumen/W      20 lumen/W  50-100 lumen/W

Se Luminous_efficacy för en översikt av effektiviteten (verkningsgraden) av olika ljuskällor. Se länk 2 och Plasma_lamp för information om plasmalampor.



/Peter Ekström

Nyckelord: *miljöpåverkan [14]; uppvärmning av bostäder [2]; lågenergilampa [13]; glödlampa [23]; lysdiod [14]; lysrör [10]; halogenlampa [3]; verkningsgrad [26]; #ljus [63];

1 http://www.blewbury.co.uk/energy/lighting.htm
2 http://www.nyteknik.se/nyheter/energi_miljo/energi/article3079117.ece

*

Ljud-Ljus-Vågor [11883]

Fråga:
Ställer mig tveksam till svaret på fråga 11865. Att knäppet i bakplåten hörs är väl snarare en effekt av att ljuset värmer upp luften som utvidgas och "knäpper" bakplåten. Blixten måste nämligen hållas nära bakplåten. En blixt som hålls längre ifrån (och där försås inte ljuset sprids åt sidorna) skulle inte ge något knäpp.
/Marcus P, Örebro

Svar:
Din kommentar till fråga 11865 har väckt tvivel i mitt sinne, så jag gjorde ett experiment för att undersöka saken. Jag sotade med en stearinljuslåga ena sidan av en bit aluminiumfolie av en handflatas storlek. När man sotar aluminiumfolie på detta sätt, måste man hela tiden hålla den i snabb rörelse, annars smälter den. Vad kan vi nu vänta oss om vi bränner av en fotoblixt mot folien?

1. Knäppet uppstår genom impulsöverföring (ljustryck). Ljus som reflekteras överför dubbelt så mycket impuls som ljus som absorberas. Det bör knäppa starkare när man blixtrar mot den speglande ytan.

2. Knäppet uppstår genom snabb uppvärmning af luften. En aluminiumyta reflekterar ungefär 95 % av ljuset, medan sotet absorberar nästan allt ljuset. Det bör knäppa starkast när man blixtrar mot sotet.

Det hördes ett starkt knäpp när jag blixtrade mot sotet, men inget alls när jag blixtrade mot den blanka ytan. Det är alltså 2. som gäller, alltså att din invändning är helt befogad. Sotet värms upp av den snabba blixten. En del av värmen överförs till luften som expanderar. Det knäpper.

Vi tackar för din kommentar, och ska nu rätta till de ställen där vi givit vilseledande svar.

Tillägg 30/11/2016: (LPE)

Här är en video med ett liknande försök:

Se Crookes_radiometer , bilden nedan, för detaljerad förklaring av ett liknande försök.



/KS/LPE

Se även fråga 11865 och fråga 20427

Nyckelord: knäppande aluminiumfolie [2]; #ljus [63]; *vardagsfysik [64];

*

Ljud-Ljus-Vågor [9897]

Fråga:
Ljushastigheten tycks ju vara en konstanternas konstant, men finns det någon bra anledning till att just ljushastigheten blivit till någonting så absolut?
/Jimmy K, Martin-Kochgymnasiet, Hedemora

Svar:
Det här är en fundamental och intressant fråga. Ljushastigheten (c) beror på de elektriska och magnetiska egenskaperna hos vakuum. Vi har sambandet:

c2 = 1/(eo mo)

Alltså, ljushastigheten i kvadrat är lika med den inverterade produkten av dielektricitetskonstanten för vakuum och den magnetiska permeabiliteten för vakuum. Dessa konstanter har samma värde hur vi än rör oss. Alltså är ljusets hastighet densamma, hur vi än rör oss. Detta är grunden för den speciella relativitetsteorin.

I den klassiska mekaniken och i vardagstillvaron kan vi addera hastigheter. Det kan man inte i relativitetsteorin.

Man kan fråga sig vad det är som ligger bakom dessa egenskaper hos vakuum. Kvantelektrodynamiken ger en viss belysning av detta. Där beskrivs ljusets utbredning som att energin producerar ett virtuellt elektron-positron par, som snabbt förintar sig, och återutsänder ljuset. Det gör att ljuset inte går oändligt snabbt, utan måste "sega" sig fram genom vakuum med futtiga 299792458 m/s.

Det var många långa ord i det här svaret!
/KS

Se även fråga 9461

Nyckelord: ljushastigheten [24]; #ljus [63];

*

Ljud-Ljus-Vågor [11135]

Fråga:
Varför bryts ljus som går från ett tunnare till ett tätare medium mot normalen?
/Ali M, hedbergska, Sundsvall

Svar:
Brytningslagen (Snells lag) kan härledas från Fermats princip som säger:

   Ljuset följer den väg som tar kortast tid

Man ställer upp ett generellt uttryck för tiden för alla tänkbara vägar. Sedan deriverar man och sätter derivatan lika med noll (för minimum). Då har vi brytningslagen.

Figuren nedan (från länk 1) ger en matematisk härledning av brytningslagen från Fermats princip. Observera att ljusets hastighet i medierna är v=c/n1 och v'=c/n2. Länk 1 innehåller även mosvarande härledning av reflektionslagen. Se även Fermat's_principle och Huygens-Fresnel_principle .



/KS/lpe

Nyckelord: Fermats princip [1]; ljusbrytning [26]; ljusreflektion [18]; #ljus [63];

1 http://hyperphysics.phy-astr.gsu.edu/hbase/phyopt/fermat.html

*

Ljud-Ljus-Vågor [10888]

Fråga:
Varför är gräset grönt?
/Veckans fråga

Ursprunglig fråga:
Varför är gräset grönt? Gräs innehåller klorofyll som absorberar solljuset. Om klorofyll huvudsakligen absorberar det gröna ljuset, varför blir då gräset grönt?
/Olle H, Västerhöjdsgymnasiet, Skövde

Svar:
Att gräset är grönt beror på att klorofyll inte absorberar det gröna ljuset. Det är ljus av andra våglängder som absorberas. Figuren nedan från Wikimedia Commons visar absorptionen för två olika sortes klorofyll. Som synes är det medelvåglängder som inte absorberas, och dessa uppfattar ögat som grönt. Se vidare Klorofyll och Chlorophyll .

Man kan använda detta faktum att växterna alltså bara kan tillgodogöra sig två våglängder (rött och blått) genom att använda speciellt avstämda lysdioder som belysning, se länk 1. Man spar alltså en massa energi genom att bara belysa med de våglängder växterna kan tillgodogöra sig, jämfört med att använda en vanlig glödlampa som till en stor del strålar i våglängder växterna inte han någon nytta av.

Här är en video från Lunds universitet om växternas färseende och användning av lysdioder i växthus:

Länk 2 är från ett företag som tillhandahåller LED-belysning för växthus.

/*fa2010_2



/Peter E

Se även fråga 16891

Nyckelord: färg/färgseende [39]; lysdiod [14]; genomskinlighet [18]; #ljus [63];

1 http://www.hydrogrowled.com/index.php?option=com_content&view=article&id=13&Itemid=17
2 http://www.professionalgrow.se/sv/belysning

*

Ljud-Ljus-Vågor [2391]

Fråga:
Varför skimrar plastfodralen till kassettband i olika färger?
/Veckans fråga

Ursprunglig fråga:
Varför skimrar plastfodralen till kassettband i reflekterat ljus?
/erik w, östersund

Svar:
Du ser denna effekt så snart du har att göra med tunna genomskinliga skikt, som t.ex. plastfodral, såpbubblor eller ett tunnt lager bensin på vattnet.

Det beror på att en del av ljuset går igenom det tunna skiktet och reflekteras, medan en del reflekteras redan i den första ytan. Detta två ljusstrålar interfererar med varandra så att vissa färger förstärks och vissa släcks ut beroende på om vägskillnaden är ett jämnt antal våglängder eller ej. Eftersom skiktets tjocklek ofta varierar i olika punkter, så ser du ett färgat mönster.

Vad händer då med fotonerna (ljuspartiklarna) som släcks ut? Försvinner de bara? I så fall skulle lagen om energins bevarande inte vara uppfylld!

Ingen fara! Ljuset (energin) kan aldrig försvinna genom interferens. Vad som sker är att man får en omfördelning av strålningen, dvs de fotoner som saknas i de mörka områdena har i stället gått ut i en annan riktning. Antalet fotoner (energin) är alltså konstant, medan fördelningen i olika riktningar ändras genom interferensen.

Vissa fjärilar åstadkommer ett nästan självlysande intryck med hjälp av fjäll av exakt anpassad tjocklek, se länk 1:



/Peter Ekström

Nyckelord: interferens [14]; *vardagsfysik [64]; #ljus [63];

1 http://animals.howstuffworks.com/insects/butterfly-colors.htm

*

Materiens innersta-Atomer-Kärnor [4383]

Fråga:
Vad är det som bestämmer färgerna på ett norrsken?
/Veckans fråga

Ursprunglig fråga:
Vad är det som bestämmer färgerna på ett norrsken?
/Pia S, Malmens friskola, Malmgerget

Svar:
Färgerna är spektrallinjer från syreatomer. I jordens magnetfält finns laddade partiklar som i spiralbanor åker fram och tillbaka mellan nord- och sydpol (strålningsbälten eller van Allen-bälten). Om en elektron från dessa bälten når tillräckligt långt ner kan det hända att den kolliderar med en syremolekyl, som kan gå sönder i två syreatomer i högt energitillstånd.

Två energinivåer är intressanta för norrskenet. Det ena har en livstid av 1 sekund, och ger grönt ljus, och syreatomen övergår till det andra tillståndet som har en livstid av 2 minuter, och ger rött ljus. Syreatomerna kan också gå ner i energi utan ljus genom kollisioner med andra molekyler. Därför uppträder norrsken bara där atmosfären är så tunn att kollisioner är sällsynta. Det är också förklaringen till att det röda ljuset syns högre upp än det gröna. Om det rör sig om ett livligt norrsken kan man se gröna norrskensstrålar som lyser till kraftigt och sedan slocknar. Kvar blir ett diffust rött sken, som långsamt dör ut.

Ibland kan ljus från kväve uppträda, men ljuset från syreatomerna dominerar. Det gröna ljuset är vanligast.

Elektroner och protoner i strålningsbältena kommer från den s.k. solvinden som består av laddade partiklar som sänds ut från solens yta. Beroende på solaktiviteten varierar solvinden ganska mycket. I de allra nordligaste delarna av Sverige är norrsken ganska vanliga. I södra Sverige är norrsken ovanliga, och förekommer bara när solaktiviteten är mycket hög. Kraftiga strömmar av laddade partiklar orsakar, förutom norrsken, även magnetiska stormar som kan störa kommunikation och el-försörjning.

Se även Norrsken, IRF, Kiruna . Bilden nedan kommer från Wikimedia Commons. Wikipedia: Polarsken och Solvind .



/KS/LPE

Se även fråga 4421

Nyckelord: norrsken [7]; #ljus [63];

*

Ljud-Ljus-Vågor [9810]

Fråga:
Gränsvinkeln för totalreflektion sägs vara då den brutna linjen bildar 90 grader med ytnormalen. Inträffar totalreflektion verkligen då eller måste infallsvinkeln vara större än detta gränsvärde?
/Sebastian J, Vasa

Svar:
Totalreflektion är ett fenomen, då ljusstrålar reflekteras i en gränsyta mellan två medier med olika optisk täthet. Om ljuset kommer från det optiskt tätare materialet, finns vid tillräckligt stor infallsvinkel inget utrymme för en bruten stråle i det optiskt tunnare mediet, och allt ljus reflekteras tillbaka in i det optiskt tätare mediet, se nedanstående figur från Totalreflexion .

q betecknar infallsvinkeln (vinkeln mot normalen). Gränsvinkeln för totalreflektion är (brytningslagen):

qc = sin-1(n2/n1)

Här antas om brytningsindexen för de båda medierna att n1 > n2 .

Totalreflektion inträffar då q > qc, gränsen är alltså då utgångsvinkeln är 90o.



/KS/lpe

Nyckelord: totalreflektion [9]; ljusbrytning [26]; #ljus [63];

*

Ljud-Ljus-Vågor [9333]

Fråga:
Hur kommer det sig att en svart yta avger mer värme än en ljus? (Jag vet att det har med svartkroppsstrålning att göra men hittar ingen förklaringsmodell som fungerar, det är lättare att förstå varför en svart yta absorberar ljus bra.)
/Mikael L, Sannerudsskolan, Kil

Svar:
I härledningen av Stefan–Boltzmanns lag (Stefan–Boltzmann_law ) om strålningen från en yta från Plancks strålningslag ingår en konstant som kan kallas spektral emissivitet e(l). Den är normalt en funktion av våglängden hos strålningen. Den har värdet 1 för en svart yta och 0 för en perfekt speglande yta.

Absorptionsförmågan hos en kropp beskrivs av den spektrala absorptansen a(l).

Kirchhoffs strålningslag (Kirchhoff's_law_of_thermal_radiation ) säger att dessa konstanter är lika:

  e(l) = a(l)

Lagen säger alltså att en god absorbator, en svart och matt yta, även strålar ut värmestrålning effektivt, medan en blank yta strålar ut mindre effektivt, se fråga 14368 .

I Kirchhoff's_law_of_thermal_radiation#Theory förs ett (ganska komplicerat) resonemang som visar att Kirchhoffs lag måste gälla. Härledbarheten har diskuterats i 150 år, men lagen är emellertid väl etablerad experimentellt.

Länk 1 är en mycket detaljerad diskussion om härledbarheten hos Kirchhoffs strålningslag: Experimenting theory: The proofs of Kirchhoff's radiation law before and after Planck.
/Peter E

Nyckelord: temperaturstrålning [29]; Kirchhoffs strålningslag [4]; #ljus [63];

1 http://www.mzwtg.mwn.tum.de/fileadmin/w00bmt/www/Arbeitspapiere/Schirrmacher_2001_1.pdf

*

Ljud-Ljus-Vågor [7809]

Fråga:
Numera anvädns s k reflexfria glasögon Hur fungerar de? Släpper de igenom UV ljus och i så fall vilken typ dvs A, B ,C?

I vissa fall anses även blått ljus skadligt för ögonen. Finns det några glasögon som filtrerar bort blått ljus och samtidigt allt UV-ljus.
/Hilma A

Svar:
Antireflexbehandling av glas innebär att man belägger glaset med ett tunt skikt av ett antireflexmaterial med annorlunda brytningsindex. Ljuset kommer då att reflekteras dels i gränsskiktet glas-antireflexmaterial, dels i gränsskiktet antireflexmaterial-luft. Om då vägskillnaden är en halv våglängd, kommer de båda reflekterade strålarna släcka ut varandra genom destruktiv interferens. För att detta ska uppfyllas helt, ställs vissa krav på brytningsindexen. Observera att antireflexbehandlingen bara funkar till 100% för en viss våglängd. Man väljer då en våglängd där ögat har högst känslighet, alltså gulgrönt ljus. Därför reflekterar antireflexbehandlad optik ljus av andra våglängder. Vanligtvis verkar den vara blåaktig.

Om en del av den reflekterade strålningen "interfereras bort" var tar den då vägen? Strålningsenergi kan inte bara försvinna. Svaret är att den transmitteras. Man får alltså genom interferens en lite större andel av ljuset att ta den önskvärda vägen genom linsen.

Se vidare Antireflexbehandling#Tunnfilmsantireflexbehandling .

Antireflexbehadlingen har knappast något inflytande på glasets egenskaper när det gäller ultraviolett ljus.
/KS/lpe

Nyckelord: interferens [14]; glasögon [2]; ljusreflektion [18]; #ljus [63];

*

Ljud-Ljus-Vågor [5528]

Fråga:
Tre polarisationsfilter
/Veckans fråga

Ursprunglig fråga:
En komplicerad fråga...... Vi har precis satt upp tre polaroidfilter, Det första och det sista är vinkelräta mot varandra (= svart) MEN det tredje som sitter i mitten är vinklat ca 45 grader och då släpps ljuset igenom........ Hur blir det så???? vrids ljuset på något konstigt sätt????
/Jonas P, Rudbecksgymnasiet, Tidaholm

Svar:
Fall 1: Vi analyserar opolariserat ljus med det första filtret. Hälften kommer igenom, och nu är det linjärpolariserat (Figur 1 nedan). Eftersom det andra filtret är orienterat vinkelrät, kommer inget ljus igenom det (Figur 2).

Fall 2: (Figur 3) Samma resonemang för det första filtret. Det andra filtret är nu orienterat i 45o. Vi delar upp ljuset från första filtret i två vinkelräta komponenter i 45o och 135o med amplituden 1/20.5 vardera. Den ena blir helt absorberad av filter 2 (som ju står i 45o), den andra släpps igenom oförändrad. Intensiteten som släpps igenon filter 2 blir då 1/2. För filter 3 blir resonemanget precis det samma. Multiplicerar vi de 3 transmissionsvärdena, får vi att (1/2)*(1/2)*(1/2) = 1/8 av ljuset kommer igenom. Här bortser vi från förluster genom reflektion.

Se även fråga 12347 och detaljerade förklaringar på engelska under länk 1 och länk 2. Figuren nedan (© Copyright 2004 Darel Rex Finley) är delvis lånad från länk 1.

I undre delen av nedanstående figur visas hur det ser ut med riktiga polarisatorer.



/KS/lpe

Nyckelord: polarisation [7]; #ljus [63];

1 http://alienryderflex.com/polarizer/
2 http://www.informationphilosopher.com/solutions/experiments/dirac_3-polarizers/

*

Ljud-Ljus-Vågor [1553]

Fråga:
Varför är himlen blå och solen gul? Vad är färg? Varför blir det färger i såpbubblor? Hur uppstår färg?
/Teofil s, Falkenbergs gymnasie, Falkenberg

Svar:
Himlen är blå därför att solljuset sprids av luftmolekylerna, och det blå ljuset sprids effektivast. Ett mera detaljerat svar har du i fråga 13368 .

Solen är inte gul om solstrålningen inte påverkas av atmosfären. Experiment: Håll ett vitt papper i solen när den står högt på himlen. Vilken färg får pappret? Ovansidan av sommarmoln är vita eftersom de reflekterar det vita solljuset effektivt.

När solen står lågt på himlen kan den uppfattas som gul eller orange. Det beror på att solljuset har lång väg genom atmosfären och att det kortvågiga (blå) ljuset sprids mer (se igen fråga 13368 ). Tar vi bort blått ljus så ser solen mer gul/orange ut, se vänstra cirkeln i nedanstående figur där man har minskat det blå inslaget lite grann i en vit skiva. När solen står högt påverkas ljuset mycket lite av atmosfären, och solen ser vit ut. Problemet är att solen lyser så starkt att man inte kan titta på den: titta aldrig direkt på solen utan skydd - du kan skada ögat permanent mycket snabbt!. Om man är mycket försiktig kan man se solen även när den står högt på himlen genom ett tunnt moln. Molnet fungerar som ett gråfilter: det absorberar alla våglängder lika eftersom vattendropparna i molnet är stora jämfört med ljusets våglängd. Man kan då se att solen faktiskt är närmast vit. Se även länk 1.

Den färg vi uppfattar beror dels på strålningens våglängdsfördelning och dels ögats känslighet i de tre våglängdsband som tapparna är känsliga för, se fråga 13824 . Det mänskliga ögat har i stort sett utvecklats med solljus. Det vi uppfattar bör då nästan per definition vara vitt ljus.

Varför ritar man solen gul på en teckning? Gör man inget alls, ser det ut som man glömt något. Därför använder man den gula färgen, den verkar ljusast.

Färg är en egenskap hos ljuset, nämligen dess våglängd. Ljuset är ju en vågrörelse. Rött ljus har lång våglängd, blått ljus har kort våglängd, och andra färger däremellan. Man kan också ha en blandning av olika våglängder (färger). Experiment: Låt solljuset lysa genom ett glasprisma.

I en såpbubbla reflekteras (speglas) ljuset både i ytter- och innerväggen. De reflekterade strålarna kan samverka eller motverka varandra, beroende på bubblans tjocklek och ljusets våglängd (färg). Fenomenet kallas interferens.

Färg kan uppstå på många olika sätt.

Molekylspridning (blå himmel)

Absorption (saften är röd därför att bara det röda ljuset kommer igenom, det andra ljuset absorberas)

Emission (doppar du en tråd i saltlösning, och sticker in den i en ljuslåga blir det gult därför att natriumatomer sänder ut gult ljus)

Interferens (såpbubbla)

Detta är några exempel.  



/KS/lpe

Nyckelord: färg/färgseende [39]; blå himmel [12]; #ljus [63];

1 http://www.universetoday.com/18689/color-of-the-sun/

*

Ljud-Ljus-Vågor [176]

Fråga:
Hur uppkommer ett absorptionsspektrum?
/Veckans fråga

Ursprunglig fråga:
Beskriv olika typer av spektrum såsom band-, linje-, absorbtions-, emissions- och röntgenspektrum.
/

Svar:
Ljuset som kommer från atomer, molekyler eller atomkärnor är "budbärare" som för med sig information om dessa mycket små kroppar som vi inte kan observera direkt. Ljus utsänds från dessa mikroskopiska kroppar när de "gör sig av" med energi.

Ett spektrum får man när man delar upp ljuset efter våglängd eller frekvens. Ett kontinuerligt spektrum innehåller ljus av alla våglängder (t.ex. temperaturspektrum). Ett diskret spektrum består av ett ändligt antal ljusa linjer (t.ex. emissionsspektrum).

Bandspektra är typiskt för molekyler och beror på att molekyler kan både vibrera och rotera, se bilden nedan.

Tydliga linjespektra kommer från atomer. Dessa som alltid är sfäriska kan varken rotera eller vibrera. Det enda sättet för en atom att ha extra energi är att elektronerna får högre energi.

I emissionsspektra ser man ljusa linjer medan man i absorptionsspektra skickar vitt ljus genom ett prov. Man ser då mörka linjer. Se nedanstående bild. Det kontinuerliga spektrat till vänster (riktning 1) kallas temperaturspektrum (se temperaturstrålning ) eftersom utseendet endast beror av den utsändande kroppens temperatur. Om man tittar på lampans temperaturspektrum genom ett gasmoln, så kommer vissa våglängder att absorberas. Vi får ett absorptionsspektrum (riktning 3). Om man i stället tittar på gasmolnet från sidan, så kommer man att se ljusa linjer - ett emissionsspektrum (riktning 2). För ett visst gasmoln är våglängderna för linjerna samma i emission som absorption förutom att vissa linjer ibland saknas i absorptionsspektrat, se fråga 15042 .

Man kan fråga sig varför man får mörka linjer i riktning 3 när de våglängder som absorberas i gasmolnet sänds ut igen. Anledningen är att de sänds ut i alla riktningar (t.ex. i riktning 2), så det kommer färre fotoner av de absorberade/utsända våglängderna i riktning 3.

Röntgenspektra har mycket kortare våglängd än synligt ljus. Dessa spektra består av två komponeneter: dels kontinuerligt, dels ett karakteristiskt spektrum.

Studera: Studera olika spektra från gasurladdningslampor med hjälp av spektroskop. För att se absorptionsspektrum kan Du på motsvarande sätt studera solljuset.



/Gunnar O/Peter E

Nyckelord: spektrum [11]; temperaturstrålning [29]; #ljus [63];

*

Ljud-Ljus-Vågor [4606]

Fråga:
En elev frågade mig idag om inte regnbågen egentligen är rund, men att vi inte ser hela? Finns det runda regnbågar? Kan man se flera regnbågar på samma gång?
/Björn M, Påskbergsskolan, Varberg

Svar:
Sitter man uppe på ett berg eller en radiomast och har tur med vädret, kan man få se regnbågen sträcka sig runt hela cirkeln. Observatörens skugga är i centrum.

Vi ser en regnbåge eftersom ljuskällan (solen) är liten, i princip en punktkälla. Regnbågen är alltså en avbildning av en puntkälla i en massa vattendroppar. En utspridd ljuskälla skulle inte ge en bra uppdelning i färger. Detta är analogt till att man använder en smal spalt i ett spektroskop.

Regnbågen är alltså en cirkel med centrum 180o från solen. Vanligen ser man två regnbågar, en starkare inre (vid 42o, se nedanstående bild från Wikimedia Commons ) och en svagare lite längre ut. Mellan dem är det mörkt (Alexanders band). Innanför den inre är det påtagligt ljust. Slå på regnbåge i Nationalencyklopedin eller titta på Wikipedias artiklar Rainbow och Regnbåge .



/KS/lpe

Nyckelord: regnbåge [6]; #ljus [63];

*

Ljud-Ljus-Vågor [3545]

Fråga:
Vad består ljuset av ? varför har det alltid konstant hastighet oavsett utgångshastighet? När uppmättes ljushastigheten första gången? Av vem? Hur gjorde han?
/Erik Å, Lars Kaggskolan, kalmar

Svar:
Ljuset är en så kallad elektromagnetisk vågrörelse, men när ljuset sänds ut och absorberas sker det i bestämda energipaket som kallas fotoner.

Att ljusets hastighet alltid är konstant (i vakuum) är ett antagande (så kallat postulat) man gör i den speciella relativitetsteorin. Det kan därför inte förklaras inom ramen av relativitetsteorin. Eftersom teorin tycks stämma väl med mätresultat, får vi helt enkelt konstatera att naturen tycks vara så. Man kan också undersöka saken med den nyare elektromagnetiska teorin (kvantelektrodynamiken). Där beror ljusets hastighet på vissa intressanta egenskaper hos vakuum.

I många fall kan det vara svårt att säga vem som var först, men inte i detta fallet. Det var en dansk astronom som hette Ole Römer. Året var 1676 och platsen Paris. Han mätte tidpunkterna för förmörkelser av jupitermånen Io, alltså tidpunkterna då Io går in i och ut ur Jupiters skugga. Han fann en periodisk variation med perioden lite mer än ett år. Avståndet till Jupiter varierar ju när jorden går i sin bana runt solen. Han insåg att den periodiska variationen kunde förklaras om ljuset hade ändlig hastighet, och han räknade ut ungefär rätt hastighet. Tänk att detta var över 300 år sedan. Ganska genialiskt, eller hur?

Slår man på ljus i Nationalencyklopedin blir man lätt förvirrad. I texten beskrivs Ole Römers metod på ett sätt, och i bildtexten på ett helt annat sätt. I själva verket är det två oberoende metoder. Han använde sig av båda, och eftersom de gav samma resultat, kunde han känna sig ganska säker på att han hade rätt. Mera kan du läsa om det här: The Speed of Light .
/KS/lpe

Se även fråga 2930 och fråga 3534

Nyckelord: #ljus [63]; ljushastigheten [24];

*

Energi [2439]

Fråga:
Hur kan vissa saker bli självlysande?
/Anna K, Peder skrivare, varberg

Svar:
Med "självlysande" menar vi vanligen att något skickar ut "kallt ljus", alltså utan att glöda. Med ett fint ord kallas fenomenet luminiscens (luminiscens ). Beroende på varifrån energin kommer delas den in i olika typer.

När vi talar om fluorescens (fluorescens ) är den inkommande energin ljus med kortare våglängd än det utsända. Exempel: De starkt lysande kläder, som man ser på vägarbetare, lysrör och vita lysdioder.

Kommer energin från en kemisk reaktion kallar vi det kemoluminiscens. Exempel: Oxidation av gul fosfor.

En speciell typ av kemoluminiscens kallas bioluminiscens (bioluminiscens ). Då sker reaktionen i ett biologiskt system. Exempel: Eldfluga, mareld, honungsskivling (en svamp).

Elektroluminiscens (elektroluminiscens ) tar energin från en elektrisk ström. Exempel: Lysdiod

Fosforescens (fosforescens ) talar vi om när energin kommer från tidigare belysning. Exempel: Det finns strömbrytare, som lyser en stund efter att man släckt ljuset.

Självlysande siffror lyser tack vare radioluminiscens. Då kommer energin från ett radioaktivt preparat, förr radium, numera tritium.

Det finns en del andra luminiscensfenomen, men de här är de viktigaste, se Luminescence .
/KS/lpe

Nyckelord: luminiscens [5]; fluorescens [6]; #ljus [63];

*

Ljud-Ljus-Vågor [793]

Fråga:
Vart tar ljuset vägen när man släcker lampan ?
/Veckans fråga

Ursprunglig fråga:
Om man har ett ljusisolerat rum med en lampa i, var tar ljuset vägen när man släcker lampan ?
/Sandra A, Ölyckeskolan, Löberöd

Svar:
Låt oss först titta på "ett vanligt rum". Ljuset strålar hela tiden ut från lampan och träffar väggarna där det mesta "sugs upp" av väggen och resten reflekteras. Om man släcker lampan så dröjer det inte länge förrän allt ljuset "sugits upp" av väggarna. Energin i ljuset övergår i värmeenergi i väggarna.

Du säger ljusisolerat rum och med det menar Du säkert ett rum som har speglar på alla väggarna och även på golvet och i taket. Om man släcker ljuset i ett sådant rum så skulle ljuset studsa fram och tillbaka för all framtid om speglarna vore helt perfekta. I verkligheten är de inte det – vid varje reflektion "sugs" lite av ljuset upp och det dör ut efter hand.

Observera att det ljus du ser (det som hamnar i ögat) försvinner i ögat. Det betyder att även om du har perfekta speglar så kan du inte observera ljuset utan att förstöra det,

Ljus som färdas ut i världsrymden kommer att fortsätta i oändig tid om det inte träffar på något. Kommer rymden då att bli "ljusare"? Nej, pga universums expansion gör den inte det. Se vidare Olbers paradox .
/GO/lpe

Nyckelord: *vardagsfysik [64]; #ljus [63];

*

Energi [1129]

Fråga:
Solenergi
/Veckans fråga

Ursprunglig fråga:
Kan man använda solenergi över hela världen? Vilka olika typer av solfångare finns det? Vad kostar det?
/Per L, Näsbydalskolan, Täby

Svar:
Solenergi är energi som ytterst härrör från kärnprocesser i solens inre och som når oss i form av solstrålning. Se solens energiproduktion om hur denna energi produceras.

Solljusets betydelse för såväl naturliga processer på jorden som mänsklig verksamhet visar sig i att nästan all energiomsättning på jorden härrör från solen. Det enda andra energiutbytet alstras av radioaktiva ämnen, som driver kärnkraftverk (kärnenergi) samt svarar för en stor del av värmen i jordens innandöme (geotermisk energi).

Detta betyder att ca 95 % av världens energianvändning direkt eller indirekt utgörs av solenergi, till största delen lagrad i en eller annan form. Hit hör således de fossila bränslena, som utgör solenergi som lagrades för hundratals miljoner år sedan, biomassa som lagrats i år eller tiotals år i form av träd och annan växtlighet, samt jordvärme, där sommarens solinstrålning används för att producera varmvatten även på vintern. Vattenkraft är också lagrad solenergi - solinstrålningen skapar moln som ger regn i höglänta områden. Även vind- och vågenergi är ytterst solenergi eftersom rörelsen i atmosfär/hav även drivs av solinstrålningen. Dessa energislag behandlas vanligtvis dock för sig. När vi i dagligt tal talar om solenergi avser vi vanligen den direkta omvandling av solstrålningen till el och värme som sker i solceller och solfångare eller i stor skala i solkraftverk.

För en uthållig framtid behöver vi energisystem som endast bygger på förnybara energikällor solstrålning, vatten i rörelse, vind, biomassa och geotermisk energi. I ett sådant system spelar även väte en viktig roll som bränsle. Väte kan framställas med hjälp av sol-el eller på fotokemisk väg.

Solenergin är en enorm resurs. Solen utstrålar årligen cirka 3.5*1018 TWh varav 7.5*108 TWh når jordytan. Av detta omvandlas knappt 0.06% i fotosyntesen och tas till vara av jordens växtlighet som energi i biomassa. Även den del av solenergin som omvandlas till vindar är mindre än en procent. Potentialen för direkt solenergianvändning är därför stor jämfört med dessa båda andra förnybara energikällor.

Världens kända och ekonomiskt utvinnbara tillgångar av olja och gas, liksom uranreserverna för dagens typ av kärnreaktorer, motsvarar den solinstrålning som når jordens yta under bara några dagar. Reserverna av kol motsvarar några veckors solinstrålning.

Den energi som solljuset innehåller kan direkt användas på i princip tre olika sätt:

1 Solvärme. Man använder solfångare som värmer vatten (eller luft) som används för att ge varmvatten eller för att värma bostäder. Detta är vanligt i medelhavsområdet där det finns mycket solsken. Eftersom behovet av uppvärmning av bostäder är begränsat används solfångarna där mest för att producera varmvatten, se nedanstående figur.

2 Solceller. I dessa celler omvandlas solenergin direkt till elektrisk energi. Många solceller brukar monteras tillsammans och bildar solpaneler. Sådana används bland annat för att förse satelliter med energi. De används även för andra tillämpningar där kabeldragning skulle vara alltför dyrt. Se vidare länk 1. Om elektriciteten inte används omedelbart måste den lagras i batterier. En nackdel med solceller är att de inte fungerar på natten och fungerar dåligt när det är molnigt. En solcell är alltså konstruerad precis som en lysdiod (se fråga 1677 ) men körs så att säga baklänges så att ljus in ger ström ut.

3 Solkraftverk. Man koncentrerar solstrålningen med hjälp av speglar på en vattenfylld behållare. Värmen får vattnet att koka, och ångan leds till en turbin som driver en generator som ger elektrisk ström. En sådan här anläggning kräver mycket solljus så den lämpar sig bäst för öknar i varma områden.

Priset på solenergi är fortfarande inte konkurrenskraftigt jämfört med andra energiformer utom för speciella tillämpningar.

Ta reda på. Hur är en vanlig solfångare konstruerad?

Projekt. Konstruera en enkel solfångare som kan användas för att ge varmvatten till duschen i ett fritidshus.

Se vidare Solar_power , Solar_energy , Solenergi , solenergi och Solcell .



/Peter E

Se även fråga 332

Nyckelord: solenergi [14]; energikällor [26]; #ljus [63];

1 http://science.howstuffworks.com/solar-cell.htm

*

Ljud-Ljus-Vågor [841]

Fråga:
Varför är himlen ibland blå och ibland röd?
/Veckans fråga

Ursprunglig fråga:
Hej. Om det är av samma orsak som himlen ibland är röd som den ibland är blå, borde den då inte också lika ofta vara grön, då grön ligger mellan blå och röd i färgspektrat. Vad beror annars de olika färgerna på?
/Mats L, Pauliskolan, Malmö

Svar:
Mycket intressant fråga! Svaret är att himlen kan vara grön, det är bara det att det är sällsynt. Låt oss försöka reda ut de olika färgerna:

Mitt på dagen är himlen blå. Det beror på att solljuset sprids av luften, och blått ljus sprids mer än rött och gult. Eftersom det är spritt ljus, så är det också polariserat.

Försök: Sätt på dig ett par polarisationssolglasögon. Titta på en punkt 90 grader (ett kvarts varv) från solen. Vrid på huvudet. Kan du se att himlen blir mörkare när huvudet är i ett visst läge?

När solen håller på att gå ner, så har solljuset så lång väg att gå igenom atmosfären, att det blå ljuset är helt försvunnet (spritts bort). Det enda du ser då är det röda ljuset som sprids - du har en vackert röd solnedgång.

I ett mellanläge kan man ibland se att himlen är grön, men det är, som sagt, ganska ovanligt.

Se även snackset Varför är himlen blå? .
/Peter Ekström

Nyckelord: blå himmel [12]; polarisation [7]; *vardagsfysik [64]; #ljus [63];

1 http://science.howstuffworks.com/question39.htm

*

Ljud-Ljus-Vågor [171]

Fråga:
Brytningsindex som funktion av våglängden bildar en avtagande funktion. Hur förklarar man fysikaliskt att våglängder med högre energi får lägre fart i ett genomskinligt medium än vågor med lägre energi?

Kan konstanterna i Cauchys dipersionsformel beräknas?
/

Svar:
Dispersion är enligt Nationalencyklopedin förändring av en våg (ljus, ljud, vattenvågor etc.) som är sammansatt av flera elementarvågor, var och en med given men sinsemellan skild frekvens, till följd av att de olika elementarvågorna har olika fart. Exempel: vitt ljus som i ett prisma delas upp i olika färger, regnbågen, havsvågor. För ljus är alltså dispersion en ändring i brytningsindex som funktion av våglängden, se nedanstående figur från Wikimedia Commons (Dispersion_relation ).

Det är inte helt korrekt att brytningsindex avtar som funktion av våglängden. Detta gäller för synligt ljus för nästan alla material. Går man över ett större våglängdsområde så är denna funktion omväxlande växande och avtagande.

När ljus passerar ett material växelverkar det elektromagnetiska fältet med de laddade partiklarna, främst elektroner som finns i materialet. Elektronerna sätts i svängning på grund av denna växelverkan. Brytningsindex har sitt maximum vid resonans. När man närmar sig resonansen ökar brytningsindex för att avta när resonansvåglängden har passerats.

Man kan se effekten så att ljus aborberas och återutsänds. Detta orsakar en fördröjning, dvs den effektiva ljushastigheten är lägre än i vakuum.

Man kan härleda Cauchys dispersionsformel (se Cauchy's_equation ) i en enkel modell där elektronerna är bundna med linjära fjäderkrafter.

Läs: De flesta optikböcker på högskolenivå diskuterar denna modell t ex Hecht: "Optics".


Nyckelord: ljusbrytning [26]; #ljus [63];

*

Ämnesområde
Sök efter
Grundskolan eller gymnasiet?
Nyckelord: (Enda villkor)
Definition: (Enda villkor)
 
 

Om du inte hittar svaret i databasen eller i

Sök i svenska Wikipedia:

- fråga gärna här.

 

 

Frågelådan innehåller 7624 frågor med svar.
Senaste ändringen i databasen gjordes 2022-05-21 17:33:39.

 

** Frågelådan är stängd för nya frågor tills vidare **


sök | söktips | Veckans fråga | alla 'Veckans fråga' | ämnen | dokumentation | ställ en fråga
till diskussionsfora

 

Creative Commons License

Denna sida från NRCF är licensierad under Creative Commons:
Erkännande-Ickekommersiell-Inga bearbetningar
.